.
.
Math Central - mathcentral.uregina.ca
Quandaries & Queries
Q & Q
. .
topic card  

Topic:

number

list of
topics
. .
start over

726 items are filed under this topic.
 
Page
1/1
The next rational number 2022-01-14
From Sophia:
what comes after 3 1/2?
Answered by Penny Nom.
Simplifying a complex expression 2021-12-15
From Grace:
Write in the form. a+bi, -6-7i+8i^2+4i3-5i^4
Answered by Penny Nom.
UFO speed in MPH from nearest galaxy in one year 2021-05-22
From Joe:
If the next closest galaxy is 15 quadrillion miles away and there are 8760 hours in one year; how fast would a UFO have to fly to travel from the next galaxy, in MPH in one year ?
Google calculator gives this result: 1.7123288e+12
I don't understand what the "e+12" means.
How is that pronounced, ex. trillion, quadrillion etc..?

Answered by Harley.
Multiplying and adding two numbers 2021-02-06
From Yvonne:
Two numbers multiplied to get -13 and added to get -12
Answered by Harley Weston.
Simplify 2021-02-05
From Leslie:
To simplify improper fractions, do you convert it to a mixed number or leave as an improper fraction?

My daughter had a quiz in which she was supposed to simplify her answers. So, for example, on one question the answer she got was 11/10 and on another question she came up with 17/12. The teacher marked these as incorrect because they were mixed numbers. Her teacher wrote on her paper, "**name**, I even gave this back to you and told you to go through and simplify your answers!" We (her father and I) feel that the teacher wanted her to convert her answers, not simplify them but I haven't been able to find a definitive answer online. So, does simplifying improper fractions mean converting them to mixed numbers? Thank you for your help!


Answered by Penny Nom.
10^1,000,000 2020-12-18
From G:
Looking for the name of a really big number. its a 1. with a Million zeroes after it.
Answered by Harley Weston.
0^(a+bi) 2020-12-03
From Douglas:
I realize raising 0^a = 0 if a>0 and undefined if a<=0.

If have read that 0^bi is undefined for all b.

What I don't understand is why 0^(a+bi) = 0 if a and b are not equal to zero.

Is this purely by definition or is there a logical reason why this is the case?
(I have taken Complex Analysis, so have a fairly good understanding of complex numbers.)

Answered by Penny Nom.
Five rational numbers between -4 and -3 2020-07-28
From Priyanshi:
Five rational numbers between -4 and 3
Answered by Penny Nom.
Three numbers have a sum of 45 2020-05-06
From Jayden:
Three numbers have a sum of 45. The greatest of the 3 numbers is 2 greater than the least number. What are the numbers? *
Answered by Penny Nom.
Four digit numbers starting with 6 2020-04-22
From Judy:
I need to generate a list of 4 digit numbers using 0-9 where the first digit is 6 and no numbers can be repeated
Answered by Penny Nom.
A 5 digit number 2020-04-04
From Rajesgy:
Guess the 5 digit number
Hint:
Number is divisible by 2
Average of all numbers is 5
Last 2 digit is divisible by 7 and the number is less than 50000
Only one odd digit

Answered by Penny Nom.
Six-digit telephone numbers 2020-03-12
From Lulamile:
What is the probability that a six-digit telephone number has no repeated digits? The telephone number cannot begin with a zero
Answered by Penny Nom.
Investigating y = (-2)^x 2020-01-13
From Gonzalo:
This is not precisely a maths question, but it is formulated based on my maths curiosity. Fidgetting with my new graphic calculator, I started graphing things and had the idea to graph $y=(-2)^x.$ The result surprised me, and I thought a little bit about it, stored it on the back of my brain, and promised myself to look deeper into it someday.
Answered by Harley Weston.
What is the smallest 4 digit number? 2020-01-10
From Ullas:
What is the smallest 4 digit number?
Answered by Penny Nom.
Two digit numbers 2019-12-03
From Lacy:
Using the digits zero through nine, how many two-digit numbers can be formed,which do not have a four or five?
Answered by Penny Nom.
The divisors (aliquot parts) of a positive integer 2019-10-31
From Kenneth:
Is there a fast and simple method that can be used to determine the aliquot parts, whole, positive numbers that can divide into another whole, positive number?
Answered by Harley Weston.
Even 5 digit palindromes 2019-10-24
From daniel:
How many palindromes of 5 digits are even?
Answered by Penny Nom.
Six digit numbers from 4 digits 2019-07-30
From Tab:
How many six digit combinations can be made from the numbers zero, two, five, eight? With repetition of numbers
Answered by Penny Nom.
Two numbers 2019-05-03
From fds:
The difference of two numbers is 2, and their sum is 4. What are the two numbers?
Answered by Penny Nom.
A repeating decimal divided by a whole number 2019-05-01
From Emily:
How do you divide repeating decimals by whole numbers?
Answered by Penny Nom.
A word problem 2019-02-24
From Briana:
When 4 times the number increased by 30, the answer is the same as when 70 decreased by the number. What was my number?
Answered by Harley Weston.
Adding and multiplying two numbers 2019-02-13
From katie:
what 2 numbers add to get 7 and multiply to get 2?
Answered by Penny Nom.
Numbering pages 2019-01-20
From chungus:
It takes 852 digits to number the pages of a book consecutively. How many pages are there?
Answered by Penny Nom.
Non-repeating decimals 2018-12-28
From Grace:
What is non repeating decimal
Answered by Penny Nom.
Four digit numbers from 0, 1, 2 and 3 2018-11-30
From A parent:
how many four digit numbers can be formed using nos.0,1,2,3 if repetition is allowed?
Answered by Penny Nom.
What was the original number? 2018-11-18
From jaydaleigh:
Mark was thinking of a number. Mark adds 11 to it, then doubles it and gets an answer of 40.5. What was the original number?
Answered by Penny Nom.
Compatible numbers 2018-09-19
From Jessica:
Hello, I'm having trouble helping my daughter she is in 5th grade and I tell you her math is so much different then the math I learned in school and I been out of school for quite some time so it's a little hard for me.
Anyway her math is very confusing for me they are saying use compatible numbers to find two estimates.
I have no idea what that even mean I tried using Google but still don't get it . The problem is 12 divided by 478 using compatible numbers to find two estimates.... please help

Answered by Penny Nom.
The sum and difference of two numbers 2018-09-17
From Feni:
The sum of two numbers is 4. The difference of the two numbers is 2. What are the two numbers?
Answered by Penny Nom.
Predicting the number of fish in a lake 2018-09-16
From April:
A researcher catches 30 fish in a pond, attaches a marker to their fin and releases them. The next day she places a large net in the water and catches 9 fish. 3 have the marker attached to their fin. Can she predict the number of fish in the pond???
Thank you

Answered by Penny Nom.
The sum and difference of two numbers 2018-08-30
From Saranya:
Hi,

The question is to find the numbers from sum and difference.
The Sum = 14 and the difference = 2

I found in this web site some useful answers. but my kid is going second grade only. She don't know how to do a division or multiplication.
So can you please explain on how to find the answer using only the Addition and Subtraction.

Thanks in advance.

Answered by Penny Nom.
Creating a number sentence 2018-06-29
From trish:
First I have to solve the problem. Then write a number sentence for the problem. The problem: By installing a $120.00 thermostat that reduces the temperature setting at night a family hopes to cut its annual fuel bill for heating oil by 8%, and recover the cost of the thermostat in fuel savings after 2years. What was the family's annual fuel bill before installing the thermostat? Thanks can't figure this one out!
Answered by Penny Nom.
Three consecutive whole numbers 2018-06-22
From whaha:
the sum of three consecutive whole numbers divided by 4 is equal to 6. find the numbers.
Answered by Penny Nom.
The product of two numbers 2018-06-19
From amie:
the product of two numbers is 108.when the larger number is divided by the smaller number the quotient is 3. What are the number.tnx
Answered by Penny Nom.
The number of terms in an arithmetic sequence 2018-06-15
From Don:
how many terms in arithmetic sequence are there if the first term and the last term are 3&59 respectively in common difference is 4?
Answered by Harley Weston.
An equation with rational coefficients 2018-05-26
From Anagh:
If x and y are rational number and
(x+y) + (x - 2y)√2 = 2x -y + (x-y-1) √5

Then find the value of x and y.

Answered by Penny Nom.
An unknown number 2018-05-10
From David:
Sam chose the number 35 and made up these clues."my number is odd. It is a multiple of 5. It's other factor is greater than 5. My number is less than 40. What is my number and what is it's other factor? Are Sam's clues correct?
Answered by Penny Nom.
Gauss' Addition of whole numbers. 2018-04-30
From Brad:
I found this on your site. Question: what is the sum of the first 100 whole numbers?? Is there a different formula if the numbers begin at a number other than one? For example What is the series I want to add is goes from 7 - 53?
Answered by Harley Weston.
What is small four-digit whole number? 2018-04-15
From Rajesh:
What is small four-digit whole number?
Answered by Penny Nom.
The sum and difference of two numbers 2018-03-13
From samima:
Two numbers have a difference of 0.85 and the sum 1.What are the numbers?
Answered by Penny Nom.
Some 5 number sets 2018-03-02
From Sarah:
Hello! My question is:
If I have 5 columns of numbers, and I want to create all possible combinations (or is it permutations?) How can I do that? For example all numbers in each column may not appear in any other column. so:
column 1: 1,2,3,4,5
column 2: 6,7,8,9,10
column 3: 11,12,13,14,15
column 4: 16,17,18,19,20
column 5: 21,22,23,24,25

how may I determine all the possible ways I can get sets of 5 out if this? Thanks!

Answered by Penny Nom.
Simplify 1/2^i 2018-01-28
From Deepak:
How do I simplify this complex equation:

z=1/(2^i)

Answered by Penny Nom.
The sum and difference of two numbers 2018-01-25
From Ali:

Hello,
I was looking at the original question that was posted and answered by
your team:
http://mathcentral.uregina.ca/QQ/database/QQ.09.07/s/donna1.html

I used the 7 and 4 as example and not looking for 39 per original
question:

A=7
B=4

7+ 4 =11 and 7-4 = 3

We end up with 11 +3 = 14

A= 14/2 give use 7

B = how do you solve for B or 4 with out knowing anything about 3 or any other numbers ?

Thank you
Ali


Answered by Penny Nom.
The decimal form of a fraction 2018-01-12
From Tom:
Prove that the decimal representation of the quotient of 2 integers must repeat (if it does not terminate).
Answered by Penny Nom.
Compatible numbers 2018-01-02
From Natalie:
I'm in 4th grade and two weeks ago my class learned how to use compatible numbers. I was REALLY confused but I didn't want to say I didn't understand because I just wanted to go home. Tomorrow we have the Topic 7 Test and I'm stuck on a problem that requires using compatible numbers. My mom apparently hasn't heard of compatible numbers and she can't really help me too much. Here's the problem : The local library places new books in a section with 31 shelves. Each shelf fits 18 books. Use compatible numbers to estimate the number of books that the library can fit on the shelves.
Answered by Penny Nom.
Retiring athletes numbers 2017-12-18
From Robin:
I am wondering if you can explain in layman terms how to figure out how many different ways the numbers 00-99 can be arranged? The discussion that sparked this question revolved around the practice of retiring athletes numbers and how many actual number combinations there are. So if I were to use 00, 01, 02 03, 04, 05, 06, 07, 08, 09, 1, 2, 3, 4, 5, 6, 7, 8, 9, 10, 11, ....99. I say there are only 109 different numbers. I'm being told I'm wrong that there are way more than that. Please help. Thank you, Robin
Answered by Penny Nom.
27000001 2017-10-09
From Tulashiram:
If a× b =27000001, then what is the value of a & b ?
Answered by Penny Nom.
Changing a mixed number to a decimal 2017-09-23
From karla:
to change an improper fraction into a decimal 5 3/4 equals
Answered by Penny Nom.
Two digit numbers that contain 5 2017-08-11
From Irish:
How many two digit numbers contain the digit 5?
Answered by Penny Nom.
Some 4 digit numbers 2017-07-24
From Camille:
I need all of the 4 digit combinations that starts with number 3 and digits can't be repeated
Answered by Penny Nom.
Three consecutive natural numbers 2017-06-11
From Anita:
Three consecutive natural numbers are such that the square of the middle number exceeds the difference of the square of the other two by 60. Find the numbers.
Answered by Penny Nom.
4 digit codes with repeating digits 2017-06-01
From Morgan:
what are all possible 4 digit code repeating numbers? cuz i know that you have one website on it but, it doesn't repeat 2 numbers in a code.
Answered by Harley.
Five rational numbers between -2 and -1 2017-05-31
From Prince:
Five rational numbers between -2 and -1
Answered by Penny Nom.
Rational numbers between -2 and -3 2017-04-08
From Medha:
Find 12 rational numbers between-2 and -3
Answered by Penny Nom.
The sum and difference of two numbers 2017-03-20
From Olivia:
The sum of two numbers is 87 and their difference is 29. What are the two numbers
Answered by Penny Nom.
Is infinite a number? 2017-03-18
From Divyansh:
Is infinite a number? If yes why as i think that numbers are used for counting but infinite is undeterminable?
Answered by Penny Nom.
A rational number between -4 and -5 2017-01-03
From Anshika:
Write rational number between -4 and -5
Answered by Penny Nom.
Write these numbers from greatest to least 2016-12-22
From lsaiah:
write the following numbers from greatest to least

3 1/2, 7/3, 80%, 2.5, 0.94

Answered by Penny Nom.
x^2 = -16 2016-12-12
From A student:
x to the second power = -16

what number solves the equation?

Answered by Penny Nom.
A mixed number as a percent 2016-10-18
From Jayda:
how go you change a mixed number into a percent

Ex. 3 3/4

Answered by Penny Nom.
A five digit number 2016-10-12
From Massa:
Explain why any four digit number is less than any five digit number?
Answered by Penny Nom.
Four digit numbers 2016-09-08
From ning:
0000-9999 there is ten thousand 4-digit numbers. How to find the number of ways that 4 digit are the same? How to prove it by using formula?
Answered by Penny Nom.
The modulus of a complex number 2016-07-29
From Cheyenne:
There's a question on my Summer Assignment I cant figure out. Here it is:

Find the absolute Value of the complex number. -5i

Please help?

Answered by Penny Nom.
The number of sides of a polygon 2016-07-23
From Shriya:
All the angles of a polygon are either 155° or 140°.
There are twice as many angles of 155 °or 140°.
Find the number of sides of the polygon

Answered by Penny Nom.
Which term of this sequence has value 8? 2016-07-20
From Lauren:
Hi there

Which term in the sequence -2; -5/3; -4/3 ; -1; ... has a value of 8.

Since term 2 and 3 of this sequence contain fractions which can be converted to recurring decimals. What is the best way to work out the common difference here.

I do however understand that to work out the nth term of an arithmetic series, the following formula Tn = a + (n-1)d. In this series a = -2

Answered by Penny Nom.
Writing numbers in the form a/b 2016-07-17
From Edward:
How to write these numbers in form of a/b: -7, 0.175, -3.25, 37/10, -16, 1/3, 2.04
Answered by Penny Nom.
Subsets 2016-06-26
From Kats:
How Many sub sets are in set k={6,7,3}
Answered by Penny Nom.
The last two digits of a phone number 2016-04-03
From Joshua:
I want to find the last two digits of a phone number whose first eight numbers of a ten digit phone numbers are known.
Answered by Penny Nom.
A block wall 2016-03-15
From Robert:
I have a wall 40 ft. long and 4 ft. tall as well as another one that is 30 ft by 3.5 ft. tall that I am trying to build. how many 6 inch by 12 inch landscaping blocks do I need?
Answered by Penny Nom.
A word problem 2016-02-27
From KRK:
amrutha thinks of a number, multiplies it by 5 & 6, devides the result by 3 and takes away the number first thought of to get an answer 36. find out the number
Answered by Penny Nom.
A 2 digit number 2016-02-08
From Lloyd:
The sum of digits of a certain 2 digit number is 7. Reversing its digits increases the number by 9. Find the number
Answered by Penny Nom.
Which term of the series 2+7+12+...is 152? 2016-02-01
From francis:
whice term of the series 2+7+12+...is 152?
Answered by Penny Nom.
Complex numbers in standard form 2016-01-15
From Michael:
express the following complex numbers in standard form (2+3i)+(5-2i)
Answered by Penny Nom.
Sue's favourite number 2016-01-12
From Maria:
This is my son's homework. He is 7. Can you help?

Sue has a favourite number.
It is an even number.
It has 3 digits.
The digits add up to 15.

Find 10 numbers that could be Sue's number?

Answered by favourite number.
Strings of characters 2015-12-28
From Ali:
If I have an app and offer a two, three and four digits name and it can be only number, letters or both?
I want to know how many options will I have for each?
For example
the two digits: aa, 11, a1 or 1a
the three digits: 111, a2a, 222
the four digits: 1111, 2ge3 or 1234
So if I used all the letters and all the numbers but nothing starts with a zero How can I calculate them separately? Or what's the result for each?
Thank you
Ali

Answered by Penny Nom.
A three digit number 2015-11-13
From Tina:
The sum of the digits of a 3-digit odd number is 13. The digit in the tens place is 4 less than the digit in the hundreds place and 3 more than the ones digit. What is the number?
Answered by Penny Nom.
Is 22/7 bigger or smaller than 3.14? 2015-11-06
From Natalie:
Is 22/7 bigger or smaller than 3.14?
Answered by Harley Weston.
An example of an irrational number 2015-10-22
From Allison:
It ask me to find an example of an irrational number less than -5 and I don't understand what the difference from a rational number and an irrational number besides the fact that a rational number can be repeated and shown in a simple fraction and an irrational number can't be written in a simple fraction.
Can you help me?

Answered by Penny Nom.
2.236... 2015-10-13
From Ann:
2.236...
What is the most specific category of numbers does this fall into? Rational or Irrational? Does the .... mean that it repeats?

Answered by Harley Weston.
Square roots 2015-09-21
From mariana:
I have read various articles on how to find the square root of irrational numbers and every article out there seems to be very confusing.
i read you answer to LUKOW about irrational numbers and i am still quite confused. Say i want to find the square root of 326. i know that it is between 18 and 19 because 18 is the square root of 324 and 19 is the square root of 361 im just very confused about the rest of the process. Please help! ( if possible i would appreciate two examples. thanks)

Answered by Penny Nom.
A two digit number 2015-08-21
From Cameron:
Write the 2 digit number that matches the clues.

My number has a tens digit that is 8 more than the ones digit. Zero is not one of my digits.

My number is ________.

Answered by Penny Nom.
4 digit strings using the digits 1, 2, 3 and 4 2015-06-14
From Gary:
What is the number of possible combinations using the following fact pattern?
A. Each combination must have 4 numbers.
B. The possible numbers to choose from are 1, 2, 3, 4
C. The numbers can be included in the combination more than once. For example, it can be 1111.

Is it possible to generate a schedule of the possibilities?

Answered by Penny Nom.
Is 22/7 is an irrational number? 2015-06-07
From Sayak:
Is 22/7 is an irrational number
Answered by Penny Nom.
A two digit number 2015-06-05
From Madison:
Trini took some photos. The number of photos she took is a two digit number. The sum of the digits is 11. The tens digit is 3 more than the ones digit. How many photos did Trini take?
Answered by Penny Nom.
n^2 is a multiple of 100 2015-03-30
From Rahul:
I have to prove that n^2 is a multiple of 100 is necessary or Sufficient condition (or both) for n being multiple of 10
Answered by Penny Nom.
The product of a 2-digit number and a 3-digit number 2015-02-06
From Nathaniel:
The product of a 2-digit number and a 3-digit number is about 50 000 what are the products
Answered by Penny Nom.
Mutually prime composite numbers 2015-01-09
From nivedita:
Two composite number that is mutually prime?
Answered by Penny Nom.
The units digit of a prime 2015-01-08
From Patricia:
Every prime number greater than 10 has a digit in the ones place that is included in which set of numbers below 1 3 7 9 or 1 3 7 or 0 2 4 5 6 8 or 1 3 5 9
Answered by Walter Whiteley.
Missing digits in a phone number 2015-01-07
From Janelle:
I had a friend try and play a game on me, regarding phone numbers, so I only know the area code , and the last four digits of the phone number. Which means, there are 3 digits that I need to figure out all the possibilities to.
Answered by Penny Nom.
What is -3 squared? 2014-11-12
From Christine:
On a math test, it said "What is -3 squared?" It did NOT say "What's is (-3)squared"

The teacher's explanation is that if there are no brackets or parenthesis, you ALWAYS square the number first then do the negative, so the answer should be -9, but I can't find anywhere that confirms this.

Help please.

Answered by Robert Dawson.
A second grade word problem 2014-09-09
From Beth:
How do I explain to my second grader how to find the answer to the problem:
My tens digit is 8 more than my ones digit. My ones digit can not be zero.

Answered by Penny Nom.
A secret number 2014-09-08
From jai:
it is smaller than half of 100, it is more than 4 tens and less than 5 tens, the tens digit is two more than the ones digit , together the digits have a sum of 6..... What is my secret number?
Answered by Penny Nom.
The sum of two numbers is 52 2014-09-01
From Blake:
The sum of two numbers is 52 and the difference is 10. What are the numbers? i used to be real good at this stuff?
Answered by Penny Nom.
4821x14y is an 8-digit number divisible by 72 2014-08-06
From RAYA:
if 4821x14y is an 8-digit number divisible by 72. How many values can x and y take?
Answered by Penny Nom.
A six digit number 2014-06-30
From anupam:
hi,
my daughter asking a solution for the below question, kindly help.

Write the smallest 6 digit number wit digits ( 2,4,3,0,5,7).

plz this is an urgent requirement.

regards,
anupam

Answered by Penny Nom.
The largest odd number using the digits 08672 2014-06-24
From audrey:
what is a 5 digit greatest odd number out of this given digits? 08672
Answered by Robert Dawson.
The number of standard deviations 2014-06-15
From Dawn:
Suppose that, at a certain college, the average weight of all male students is 160 pounds with a standard deviation of 30 pounds. A certain male student weighs 145 pounds. Determine how many SD's his weight is above or below average.
Answered by Penny Nom.
How do I make fractions into decimals? 2014-05-31
From jay:
Hi,
I wanted to know how do I make fractions into decimals example 1 3/8?

Answered by Penny Nom.
The combination of 6 numbers from 1 to 355 that gives us 360 2014-05-29
From Ion:
Hi,
My name is Ion and I am trying to find the number of combinations of 6 numbers (angles) that would sum to 360 (degrees).

Thank you!

Answered by Robert Dawson.
Three digit numbers 2014-05-15
From Mudassir:
how many 3 digits numbers which at least one digit is 2 ?
Answered by Penny Nom.
A 4 digit number 2014-04-04
From LIM:
"A" is a 4 digit number formed by all the numbers from 1 to 4. When "A" is divided by 9, the remainder is the biggest possible value. What is the biggest value of A?
Answered by Chris Fisher.
Three consecutive whole numbers 2014-03-13
From Abhishek:
Three consecutive whole numbers are such that if they be divided by 5,3 and 4 respectively, the sum of the quotient is 40. What are the numbers?
Answered by Chris Fisher.
Four digit phone numbers 2014-02-25
From Tom:
Hi, I'm an online math teacher, working on lessons for my students.

This question is in their book and I can't do it...! Help!

It has to do with phone numbers, and in this problem, we are only dealing with the last 4 digits of the number.

It asks how many possible numbers we have if at least one digit repeats in the last 4 digits of the number.

Answered by Penny Nom.
A number with all the digits different 2014-02-07
From Jon:
I'm not a mathematician but just curious. Is there a name for a number of up to ten digits where each digit is different, i.e the equivalent of an isogram in linguistics? My online banking gives me a random 8-digit number each time I log on and it is rare to get one of the type I refer to. There must be calculable odds, but I'm only allowed one question!
Thanks in advance
Jon

Answered by Harley Weston.
Counting in base five using words not digits 2014-01-25
From Randy:
We all know how to count (in the base 10/decimal system) using words not numbers. For example: one, two, three, four, five, six, seven, eight, etc. However, in base 5 (for example) how would you count (in words). For sure - in base 5 the number 1 could be "one", the number 2 could be "two". However there seems to be no words to describe base 5 numbers beyond 0,1,2,3 and 4 (and perhaps 10). In base 5 the number 10 is not ten. Rather it is "five". In base 5 what word(s) describe numbers larger than 10? What words are used for 11, 12, 13, 14, 20, 21, 22, 23, 24, 30, 44,...etc. Also, consider a man and woman were married in the Gregorian year 1964. If they had an anniversary tomorrow - how long would you say they've been married in base 5 speak? In numbers their Golden Anniversary would note 200 years of marriage in base 5. How would you articulate their years together in base 5? I don't recall seeing verbiage to represent numbers in any system other than the decimal/base 10 system. Do such things exist for other systems?
Answered by Harley Weston.
A 4 digit number 2013-12-20
From swathi:
How many 4 digit numbers can be formed using the digit 3 only once but the digit 3 must be used in all numbers?
Answered by Penny Nom.
7 digit phone numbers 2013-12-08
From Sean:
Hi I was wondering how you would calculate how may 7 digit phone numbers there are with only odd digits?
Answered by Penny Nom.
Adding mixed numbers 2013-11-20
From Kathy:
1 3/4 + 1 2/3= ?

5 1/2 - 2 5/6= ?

Answered by Penny Nom.
Base 5 2013-09-30
From Zakir:
Sir I wanna know something about base 5 number system Why we use it and who use this ? and what is the purpose of it
Answered by Robert Dawson.
Two, 2-digit numbers 2013-09-24
From Tony:
Write a problem that has two 2-digit numbers.

solve your problem.

Answered by Penny Nom.
A two digit number 2013-09-04
From JEFFERSON:
The sum of the digits is 6. When the digits are interchange, the new number represented is 3 time the 10 digit of the original number. Find the original number?
Answered by Penny Nom.
What is the value of |2((i)^(1/2))|? 2013-07-22
From Delilah:
What is the value of |2((i)^(1/2))| ?
i.e. absolute value of 2 multiplied by square root of i.

Answered by Penny Nom.
What is the smallest number? (i.e. the closest number to zero) 2013-07-22
From Charlie:
What is the smallest number? (i.e. the closest number to zero)
Answered by Harley Weston.
Two numbers whose difference is 16 2013-06-08
From Johan:
Two numbers whose difference is 16 and whose sum is 120
Answered by Penny Nom.
A 2-digit number 2013-05-13
From A teacher:
Find a “2-digit number” where the sum of the 1st digit (on the Left) and the square of the 2nd digit equal the same number.
Answered by Lorraine Dame, Harley Weston.
Restricted partitions 2013-03-25
From vidya:
I am having a series of numbers eg.( 1,2,3,4,5,6,7,8,9,10,11,12,13,14,15) I can take any 5 digits eg(15,10,8,6,5) and it should not repeat and the summation should be any predefined static value . eg(44) That is (15+10+8+6+5=44) . How many summation series will result 44 ? My problem is how to find this using a formula or any other simpler automation method is there instead of checking one by one all the combinations. Plz do help me... Thnks in advance
Answered by Chris Fisher.
A 3 digit number puzzle 2013-03-11
From Bonnie:
My number has 3 digits. My number is not a multiple of 100. My number is a multiple of 50. My number is a factor of 1000. What is my number?
Answered by Penny Nom.
Write a rule for the sequence 2013-02-20
From Angelia:
Write a rule for the sequence. Then, find the unknown term. 1 3/8, 1 ¾, 2 1/8, _______, 2 7/8 Think: The pattern is increasing. Add 1/6 to find the next term. Rule: _______________
Answered by Penny Nom.
Use 9,10,6,8, and 4 to make 10 2013-01-25
From Able:
How do you calculate using BEDMAS 9,10,6,8,4 to equal 10
Answered by Penny Nom.
A valid UPC number 2012-12-22
From Jacob:
A valid UPC number , ABABABABAB- C must end in a 2 after using the rule:

3A +B+3A+B+3A+B+3A+B+3A+B+C=##2.

What would term C need to be in order to make the following UPC valid? 2741825293-C

Answered by Harley Weston.
2 1/4 x 1/8 x 1 3/4 x 12 4/9 x 3 2012-12-21
From Tony:
Ok my question

2 1/4 x 1/8 x 1 3/4 x 12 4/9 x 3

I just can't figure out the question ?

Answered by Penny Nom.
The square of any odd number, decreased by 1, is divisible by 8 2012-11-16
From bailey:
Prove that the square of any odd number, decreased by 1, is divisible by 8
Answered by Penny Nom.
Converting a mixed number to a decimal 2012-11-10
From Mahaveer:
Convert mixed fraction to decimals: 4 2/6
Answered by Penny Nom.
birdhouse in the shape of a rectangular prism 2012-10-29
From Tiffany:
Hello, I am Tiff M. from NYC.
I was solving a math problem about adding,subtracting,multiplying or dividing fractions and mixed numbers. I am not getting this question below.
* Laura is building a birdhouse in the shape of a rectangular prism. The base of the birdhouse has an area of 5 square inches. The height of the birdhouse is 2 7/12 inches. Laura calculated the volume of the bird house to be 10 7/12in. Is she correct?
If not, what did she do wrong?

Answered by Robert Dawson.
Some 4 digit numbers 2012-10-25
From samira:
How many 4-digit numbers are there in which the first and last digits are the same?
Answered by Penny Nom.
Some 6 digit numbers 2012-10-23
From Mason:
How many different 6 digit numbers can you make using the digits 1 ,2 5, 6, 7, and 9? How many of these six-digit numbers are divisible by 6?
Answered by Penny Nom.
A four digit number 2012-10-09
From cyndi:
find this 4 digit number: the tens digit is 1/3 of the hundreds digit; the 2 digit number formed by the tens and ones digits i the largest prime number smaller than 40; the sum of all 4 digits is the same as the number of face on an icosahedron ? what is the number?
Answered by Penny Nom.
Six digit numbers 2012-09-11
From Fiona:
using the numerals 0, 2, 3, 5, 7, 9, make as many six-digit numbers as you can. Rearrange them into ascending order
Answered by Robert Dawson.
A 2-digit number 2012-08-23
From patty:
Write the 2-digit number that matches the clues. My number has a tens digit that is 8 more than the ones digit. Zero is not one of my digits. My number is
Answered by Penny Nom.
Kirkman schoolgirl problem 2012-08-18
From Anthony:
need to have the number from 1-15 in 6 rounds of 3 numbers without the numbers be repeating in the same row . eg round 1
1 2 3
4 5 6
7 8 9
10 11 12
13 14 15

2nd round
1 4 7
10 13 2
5 8 11
14 3 6
9 12 15
Answered by Chris Fisher.

A cylinder is to be filled with peas. 2012-06-12
From Silje:
Hi! How can I solve the following question without the use of a calculator?

"A cylinder is to be filled with peas. It is done like this: At 12:00 o'clock you put 1 pea in, at 12:01 you put 2 peas in, at 12:02 you put 4 peas in, at 12:03 you put 8 peas in, and so on. This continues until 14:00 o'clock (two hours later), when the last peas are put in and the cylinder is full. At what time is the cylinder half full?"

Answered by Robert Dawson.
A number puzzle 2012-04-07
From Kenya:
my number is less than 25
my number of tiles will make only one rectangle
my number is odd
my number is a factor of 36

Answered by Penny Nom.
Square numbers 2012-03-17
From david:
How many square numbers are there between 0 to 100?
Answered by Penny Nom.
Figures with dots 2012-02-15
From A:
Okay so my daughter came home and had this question for homework from her teacher. I have no idea what it means HELP!!
The question shows a figure that is a square. It has 5 dots on the top, bottom and on the sides looks similar to this:
. . . . .
.       .
.       .
.       .
. . . . .
they want to know 3 more ways to show the number of dots in the figure. They have already shown one with just counting them and the other one that they showed was circling the four corner dots and then taking the dots in between with a number sentence set up like this: 4+4*(5-2) = 4+4*3= 16. They also besides giving 3 ways want us to write a number sentence and an algebraic sentence to show the number of dots in the figure with "n" sides. Please help! So confusing ! I don't know if my square turned out or not but it should be 5 across the top and bottom and on the sides and no dots in teh middle it is just the outside.

Answered by Penny Nom.
Properties of real numbers applied to subsets 2012-02-01
From Mark:
Hello - The questions that I have for you is do the properties of real numbers (such as the associative, commutative, identity, inverse, and distributive law) apply to ALL the subsets of real numbers? In other words, do all those properties work for the Natural Numbers? The Whole Numbers? And so on and so forth. I understand that they are all real numbers, but for instance: the identity is whenever you add zero to a number, you get that number back. But does that work with, say, with only the odd numbers? Zero isn't odd so can that property actually apply to JUST the odd numbers? Any consideration would be greatly appreciated!
Answered by Robert Dawson.
A two digit number 2012-01-29
From Christine:
Five times the sum of the digits of a two digit number equals the number. If the digits are reversed, it becomes nine more than the original number. What is the original number? I really don't understand this question, help me please
Answered by Penny Nom.
Triangular numbers that are square 2011-12-27
From Luke:
The “TnS” is a positive integer which is both Triangular number and Square number. For example, 36 is a “TnS” number since 36=1+2+3+4+5+6+7+8 (Triangular number) and 36=6x6 (Square number). What is the next “TnS” which is greater than 36?
Answered by Penny Nom.
The number of digits in a number base m 2011-12-22
From Jash:
Assume there is a number system of base m.

The one property of this system is: If 2 numbers written in this system, which have 'a' and 'b' as the number of digits are multiplied, then the product of the 2 numbers will have a number of digits which is a function f(a,b).

In other words, as long as the number of digits of the 2 numbers are constant, the number of digits of their product is a constant.

Find m.

Answered by Robert Dawson.
Extended real numbers 2011-12-12
From Justin:
Hi there, I was wondering does +infinity=+infinity in the extended real number system? Basically, I was wondering does +infinity=+infinity since -infinity and any extended real number (except +infinity) are less than +infinity?

Sincerely,

Justin

Answered by Robert Dawson.
Three consecutive numbers 2011-11-29
From Michael:
Find three consecutive numbers such that the second number squared is equal to the first and third added together
Answered by Penny Nom.
The units digit of a prime number 2011-11-22
From Mike:
Every prime number larger than 10 has a digit in the ones place that is included in which set of numbers below?
A. 1,3,7,9
B. 1,3,5,9
C. 0,2,4,5,6,8
D. 1,3,7

Answered by Penny Nom.
15 pigs and 6 pens 2011-11-16
From Mike:
I have 15 pigs and 6 pens, I have to put them in odd numbers, how do I do it.
Answered by Chris Fisher.
I started with Euler's identity and manipulated it 2011-11-14
From anonymous:
I started with Euler's identity and manipulated it
e^i*pi=-1
e^-i*pi=(-1)^-1
e^-i*pi=-1
e^-i*i*pi=(-1)^i
e^--pi=(-1)^i
e^pi=(-1)^i
type it in in a calculator and you get e^pi=23.1406926... and (-1)^i=0.0432139183... What did I do wrong?

Answered by Robert Dawson.
The square root of z=3+4i 2011-10-27
From dianah:
how to find the square roots of complex number, z=3+4i
Answered by Robert Dawson.
Twice an even number 2011-10-10
From Ron:
The number of dollars that Tracy has in her pocket is an odd number. Hal gave Tracy $2 more than she already had in her pocket. The number of dollars Tracy now has cannot be _________.
a) $60 b) $62 c) $64 d) 68

Answered by Penny Nom.
3-digit, daily numbers 2011-09-29
From Margaret:
I need the list/group for a raffle that would contain the combinations for a 3 digit (000-999) daily number. There would be 100 tickets sold with 10 3-digit numbers from each numerical group beginning from 000's, 100's 200's, 300's,...900's. No duplicates/replacement. What would be the list of possible combinations of each group of 100 tickets, keeping each different group/list of combinations in separate blocks of 100 tickets without duplication. So I could use 1 list of 100 this year and the next list of combos next year and so forth. Please help!

Example: 000,197,245,367,445,569,618,777,842,964

What would be the possible lists of each 100 tickets? Thank you

Answered by Penny Nom.
A number less than 100 2011-09-17
From Sharon:
I am a number less than 100. My ones digit is 6 the sum of my digit is 11. What sum am I?
Answered by p.
One factor array 2011-09-16
From Hana:
hi my brother a 5th grade have a question. this is what it say "i have only one factor array, and i am between two numbers 12 and 16. what number am i?"
Answered by Penny Nom.
A four digit number 2011-09-16
From gina:
i am having a hard time figuring this out.. i believe the answer is 3105 but the 0 is throwing me off... i am a 4 digit number between 3100 and 3200, none of my digits are even, the sum of my digits is 9, the largest digit is in the ones place, what number am i ??
Answered by Penny Nom.
A two digit number 2011-08-26
From Felicia:
Write the 2-digit number that matches the clues.
My number has a tens digit that is 8 more than the ones digit.
Zero is not one of my digits

Answered by Penny Nom.
Ben's favourite numbers 2011-08-03
From Luke:
Ben challenges you to guess his favourite numbers by using the following three clues: -Each number is a four-digit odd number. -Each is a palindrome -The digits of each number add up to 10 -None of the numbers are divisible by 5
Answered by Robert Dawson.
A rational number 2011-05-17
From owais:
according to definition of irrational number "a number which cannot be expressed in p/q form is called irrational number" square root of 2 is a irrational number but if i round off the digit correct to two decimal places so it becomes 1.41 and we can easily convert into p/q form so it is a rational number ???
Answered by Penny Nom.
Two whole numbers 2011-05-11
From yolanda:
The sum of two whole numbers is 12.If the sum of the squares of those numbers is 74,what are the two numbers?
Answered by Penny Nom.
5 digit numbers using the digits 5 and 2 2011-04-27
From sissy:
I need a list of all possible 5 digit combinations using 5 and 2 numbers can repeat
Answered by Penny Nom.
Decreasing numbers 2011-04-13
From Hadi:
A number is called a "decreasing number" if each digit in the number is less than the digit to its left. For example, 87420 is a deceasing number. How many five-digit decreasing numbers are there?
Answered by Claude Tardif.
8 3/8 - 6 1/4 2011-03-21
From lenora:
explain an error pattern in each of the following. 8 3/8 - 6 1/4 = 2 2/4
Answered by Penny Nom.
Three consecutive odd numbers 2011-03-11
From George:
Write an equation three consecutive odd numbers whose sum is 75
Answered by Penny Nom.
A two digit number 2011-03-01
From jhoanna:
the sum of the digits of a two digit number is 10. The value of the number is 16 times the units digit.Find the number.
Answered by Penny Nom.
6685 is 67.5% of the total number 2011-02-22
From Donna:
I know 6685 is 67.5%. How do I calculate the total number.
Answered by Penny Nom.
The nearest one and a half and the nearest inch 2011-02-02
From Miranda:
REASON:sarah gave the same answer when asked to round 4 seven eighth inches to the nearest one and a half and the nearest inch. explain why sarah is correct?
Answered by Penny Nom.
An even multiple of 27 2011-02-01
From parth:
the 6 digit # 63x904 is an even multiple of 27 what is X
Answered by Penny Nom.
Three consecutive numbers 2011-01-31
From parth:
name 3 consecutive numbers,each less then a 100,the smallest # is divisible by 6 the next is divisibel by 5 and the largest divisible by 4
Answered by Penny Nom.
A mixed number 2011-01-30
From Joyce:
express the fraction as a mixed or whole number 94/5
Answered by Penny Nom.
The sequence 2,3,5,5,8,7,11 2011-01-19
From taylor:
number relationship in number sequence 2,3,5,5,8,7,11
Answered by Chris Fisher and Penny Nom.
Identify each polynomial by its degree and number of terms 2011-01-10
From betty:
Write in standard form and identify each polynomial by its degree and number of terms. How do you do this?
Answered by Penny Nom.
Find all the roots 2010-12-02
From gagan:
find all the roots of z^5-3z^4+2z^3+z^2-3z+2
Answered by Stephen La Rocque and Penny Nom.
Ordering mixed numbers 2010-11-23
From kate:
what are the mixed numbers 5 7/9, 5 1/2, 5 11/18. put least to greatest?
Answered by Penny Nom.
i^i 2010-11-21
From trale:
Can we use e^ix=cosx+isinx for finding i^i like that: x= pi/2 => e^(ipi/2)=0+i then [e^(ipi/2)]^i=i^i.then we find i^i= 0,207879576.... is it true? can we give value for x for free?thank you.
Answered by Harley Weston.
What is the largest prime number? 2010-11-20
From vadali:
what is the largest prime number?
Answered by Penny Nom.
A product of 1 000 000 2010-11-19
From Sharron:
There are many pairs of numbers that when multiplied together give a product of 1 000 000. However, there is one pair, and only one pair, of whole numbers that contain no zero digits and has a product of 1 000 000. What are these two numbers?
Answered by Stephen La Rocque.
A six digit number containing no zeros 2010-11-15
From John:
Find a Six digit number containing no zeros in which the first digit is three less than the third, the third digit is three less than the fifth,and the second digit equals the sum of the last three digits.
Answered by Penny Nom.
Four-digit numbers using 0,3,4,5,6,and 7 2010-11-12
From Katy:
Using only the digits 0,3,4,5,6,and 7, how many distinct four-digit numbers exist that are greater than 4002 and less than 6732?
Answered by Stephen La Rocque.
z^5 - 3z^4 + 2z^3 + z^2 - 3z + 2 2010-11-06
From Kumar:
would you please solve this problem, related to complex numbers.

Find all the roots of :

z^5 - 3z^4 + 2z^3 + z^2 - 3z + 2

Answered by Robert Dawson and Penny Nom.
39-4+6 divided by 3 to the power of 2 = 3 2010-11-02
From Emma:
Help me and my family cannot figure this out, it the last question on my paper. I need to know how to use place the brackets to make the following problem true.

39-4+6 divided by 3 to the power of 2 = 3

we have tried for three days and cannot get it.......help!

Answered by Robert Dawson and Claude Tardif.
A three digit number 2010-09-22
From Andrew:
Both the sum and the product of its three digits are 6. The least digit is in the hundreds place and the greatest digit is in the ones place. Write it in word form:
Answered by Penny Nom.
Finding a Number Pattern 2010-09-22
From Cheryl:
Sunshine Math - 4, Jupiter 1, question 4 which is (a, b, c & d). I am wanting to know question 4d. Think about the following list of number pairs. Three is the first number of a pair, and 8 is the second. 3 - 8 4 - 11 5 - 14 6 - 17 and so forth Question d. - If a number "n" is the first number, what is the second number? I don't know how my son should answer this question. It sounds like it should be a formula but how does he get to it and how do I explain it to him. Looking forward to your help. Cheryl
Answered by Janice Cotcher.
A Squared Number That's Negative 2010-09-22
From David:
What is the only number that when it's squared becomes negative?
Answered by Stephen La Rocque.
What is -2^2? 2010-08-25
From alex:
I know that (-2)^2 is 4 but what is -2^2?
Answered by Robert Dawson and Penny Nom.
Numbers that can be formed using the digits 1,2,3,4,5,8,9 2010-07-19
From Donessa:
write the number that is 2000 more than the difference between the largest and the smallest number that can be formed using the digits 1,2,3,4,5,8,9
Answered by Tyler Wood.
777777 times 111111 2010-07-07
From Chew:
What is 777777 times 111111 without using a calculator?
Answered by Chris Fisher.
1.2731568 2010-06-16
From angel:
is 1.2731568 a natural number a whole number a irrational number or a inter
Answered by Robert Dawson.
Graphical Representation of Complex Numbers 2010-06-08
From Anas:
why do we write complex number a+ib as (a,b)?
Answered by Janice Cotcher.
11 trillion divided by 309,418,000 2010-06-05
From Dion:
I am struggling to calculate what 11 trillion divided by 309,418,000 is.
Answered by Penny Nom.
A quadratic equation with imaginary numbers 2010-06-03
From Alissa:
I am solving a quadratic equation and I got this far;
(x-4+i)(x-4-i)=0
but how do I add the imaginary numbers i know you multiply x by x and then add -4 + -4 but what do you do with the i's?

Answered by Penny Nom.
Four digit numbers 2010-05-22
From tyeisha:
Whats all the four digit numbers you can come up with using numbers 0-9
Answered by Tyler Wood.
The sum of twice a number and 6 is 8 2010-05-12
From MAE:
THE SUM OF TWICE A NUMBER AND 6 IS 8 . FIND THE NUMBER
Answered by Penny Nom.
Odd numbers with no repeated digits 2010-05-01
From jameka:
How many odd numbers can be written from the set {2,3,4,5,6} if no digits may be used more than once?
Answered by Penny Nom.
A champagne tower 2010-04-16
From gabriel:
thanks for having something like this up online. My question is how do i build a Champagne Tower that has 290 glasses and 15 stories high.
Please help me out. this is for a clarity function.

Answered by Robert Dawson.
Body measurements 2010-04-06
From Amirul:
Recently I'm proposing my research question to my teacher for my extended essay. I'm an IB student. My research question is regarding the estimation of human in buying trousers through reference of neck. What does the relation between the diameter of the neck and the diameter of the waist? I want to see how far does the estimation theory is true for different type of people with different BMI(body mass index)..

But teacher said that it is golden ratio...so nothing interesting... =( really??? But i search on net.... state that my idea seems do not have any relation with the golden ratio so far..... i just want ask you... am I able to perform in my extended essay if i continue with this research question??

Answered by Robert Dawson.
A champagne pyramid 2010-03-22
From Kathy:
I have 680 champagne glasses and my pyramid can only be 15 levels high. How many glasses need to be on each level? Is there a formula I can use?
Answered by Robert Dawson.
7 odd numbers 2010-02-19
From mike:
Hello there, Problem: find 7 odd numbers who sum is equal to 30. Hint*My professor said it's possible to add 7 odd numbers and get 30 for an answer! and he said we can use Negative and rational numbers but no decimal or fractions. So is there anyway of solving this? Please get back to ASAP! thanks.
Answered by Harley Weston and Tyler Wood.
4-digit positive numbers 2010-02-15
From marjon:
Find the sum of all the 4-digit positive numbers with no zero digit.
Answered by Penny Nom.
Mixed numbers and multiplication 2010-02-08
From Nick:
Can you explain how this works, I have been going at it for hours and have a test friday and just cant get my head around the question
3 1\4 * 1/7 * 8

I have to express in lowest terms

Answered by Penny Nom.
Three digit odd numbers 2010-02-04
From Rebecca:
Question from rebecca, a student:

how many three digit odd numbers can be formed using the digits 1,2,3,4,5,6 without using any digit more than once?

Answered by Tyler Wood.
1/a + 1/b + 1/c < 1 2010-02-01
From Mrityun:
suppose a,b and c are natural numbers such that 1/a + 1/b + 1/c < 1. Prove that

1/a + 1/b + 1/c < = 41/42.

Answered by Penny Nom and Claude Tardif.
Mixed numbers 2010-01-13
From Linh:
Perform the indicated operations:
6 1/4 + (- 2/3) x 1 1/2

Answered by Penny Nom.
A proof involving real numbers 2010-01-11
From Amper:
Let a,b is an element of real numbers, and suppose that for every x>0 we have a is lesser than or equal to b+x.
(a) Show that a is lesser than or equal to b.
(b) Show that it does not follow that a is lesser than b.
i'm feeling bad of having no idea with this, hope i you can help me. GRACIAS!!

Answered by Penny Nom.
A set of 5 numbers 2010-01-10
From Marco:
I need to generate in an Excel or other table all possible combinations for a set of 5 numbers with the following criteria:

1.- Numbers can repeat.
2.- Order does not matter.
3.- Numbers may be from 0-9
4.- The sum of all numbers is equal or less than 31.
5.- The sum of all numbers is equal or greater than 10.
6.- One of the numbers is 2.
7.- One of the numbers is 5.
8.- One of the numbers is 7.

Marco

Answered by Penny Nom.
Four digit numbers with no repeats 2010-01-02
From Daniel:
I want to know how many different numbers there would be between the numbers 0000-9999 that do not have any numbers that duplicate or repeat in the number? For example the first number would have to be 0123 and the last would be 9876 (I think) Thanks!
Answered by Penny Nom.
A 4 digit number 2009-12-15
From Lisa:
Billy Club was assigned the task of putting numbers on all the playground balls used during daily recess. Billy will number the balls using the following rules:

1. It will be a 4-digit number.

2. The digit in the thousands place will be a 1 or a 2.

3. The digit in the hundreds place will be a 2, 4, or 6.

4. The digit in the tens place will be an odd number.

5. The digit in the ones place will be greater than six.

How many balls can Billy number if he follows these rules?

Answered by Robert Dawson.
Three digit numbers 2009-12-11
From christopher:
i need a list of all 3 digit combinations with repeat of a number thnx
Answered by Penny Nom.
A number base that's not a positive integer 2009-12-11
From Nick:
Is it possible for a number to have a base that's not a positive integer? Base 1 gives a result that's meaningless but possible. I have no concept of a base 0, a negative base, or a fractional base, never mind any other base. I think there isn't any base that's not a positive integer, but, knowing that math keeps jumping ahead and sometimes has inventions before anyone knows how to exploit them, I think I'd better ask.

Thank you.

Nick

Answered by Robert Dawson and Claude Tardif.
Two digit numbers 2009-12-10
From Jim:
Can you tell me how many 2 digit numbers can be created from the following:

1118539

Answered by Penny Nom.
Three numbers that have the sum of 45 2009-12-04
From amanda:
there are three numbers that have the sum of 45.The greatest of the three numbers is 2 greater than the least number.What are the numbers?what is the formula to get the answer?
Answered by Penny Nom.
6 numbers from 12 2009-11-14
From david:
how many combinations of 6 numbers can there be using 12 different numbers without using the same numbers more than once?

2, 3, 5, 8, 9, 11, 12, 14, 17, 19, 20 & 31.

Answered by Penny Nom.
A sequence of letters and numbers 2009-11-10
From Maria:
What is the last ten letters and numbers in the following series and how do I work it out so I can explain it to an eleven year old. J1F8M1A0M1J0J1
Answered by Robert Dawson, Claude Tardif and Harley Weston.
The cardinality of the prime numbers 2009-11-07
From Justin:
Hello there, I was just wondering since the number of primes is infinite, are they equal to infinity or Aleph-null? Justin
Answered by Robert Dawson and Victoria West.
Tetrahedral numbers 2009-11-06
From rena:
consider the tetrahedral numbers 1,4,10,20, etc. Can these numbers be described as a polynomial? If so, what is the degree of that polynomial?
Answered by Stephen La Rocque.
A number puzzle 2009-11-03
From Jacqui:
Write a number sentence. Use every digit once, 7, 4, 3, 6, 5,10. Insert math symbols +, - , *,/ and end up with the number 3. Use parentheses if necessary.

Jacqui

Answered by Claude Tardif and Penny Nom.
The extended real numbers 2009-11-02
From Justin:
Hello there, I was wondering is the set of extended real numbers a closed set or an open set?

Justin

Answered by Robert Dawson.
Proof that the root of 27 is irrational 2009-10-18
From Scarlet:
How do you prove that the square root of 27 is irrational?
Answered by Victoria West.
A number between 48 and 59 2009-10-07
From nakia:
I am a number less 59 but greater than 48. my ones digit is 2 more than my tens digit.What's the number?
Answered by Penny Nom.
How many terms are there in this sequence? 2009-09-28
From tabby:
How many terms are there in this sequence?
5,1,-3,...,-111

Answered by Penny Nom.
Seven circles 2009-09-20
From Bobbi:
try to put number 1 to 7 in seven circles (one in the middle, 3 on top, 3 below) so the numbers in each row of three circles--vertical, horizontal, and diagonal -- add up to 12. Each number can be used only once.
Answered by Stephen La Rocque.
What is my number? 2009-09-18
From Hanna:
What is my number?
My number is a perfect square.
The only number in its prime factorization is 2.
My number is a factor of 32.
The sum of its digits is odd.

Answered by Penny Nom.
Cubes and squares 2009-09-16
From Stanley:
What is a three consecutive digit number like 5,6,7 , which is two less than a cube and two more than a square?
Answered by Robert Dawson.
4 number combinations 2009-09-10
From jeremy:
Here's my question... I've been playing the lottery, I know what a waste of time. I would like to know how many 4 number combinations can be derived from a set of 10 numbers, when no number can be repeated in the present string of 4 numbers being selected.
Answered by Robert Dawson and Harley Weston.
A three digit number 2009-09-04
From Mark:
How many 3 digit positive integers are odd and do not contain the digit 5?
Answered by Penny Nom.
The sum of digits of 4444^4444 2009-08-31
From SHIVDEEP:
The sum of digits of 4444^4444 is A .The sum of digits of A is B .
Find the sum of digits of B ?

Answered by Claude Tardif.
A number puzzle 2009-08-28
From Angela:
Taking 1 through 9 and using them only once subtract them to equal 33333
Answered by Chris Fisher.
30 students 2009-08-06
From Peter:
Question from Peter, a student:

Can you please help with the following questions?

(a) It is known that among any group of the three students in a class two of them are friends. Total number of students is 25. Prove that there is a student who has at least 12 friends.

(b) There are 30 students in a class. They sit at 15 double desks, each one is for two students. Half of the girls sit with boys. Is it possible to make a rearrangement so that half of the boys sit with girls?

Answered by Penny Nom.
Properties of Natural Numbers 2009-07-24
From nazrul:
If m,n,k are natural number how can I prove that (m+n)k=mk+nk. In the proof the properties of natural number should be used.
Answered by Janice Cotcher.
The extended real number system 2009-06-30
From Justin:
Hi again, thanks a lot for answering my previous question! I was also just wondering again if the extended real number system has a potential or actual infinity because it includes positive infinity as a point that exists at the end of it?

All the Best,

Justin

Answered by Robert Dawson.
Primes 2009-06-25
From Chinonyerem:
If p = 2^k - 1 is prime, show that k is an odd integer, except when k = 2. [ Hint: 3/4^n - 1 for all n >= 1.]
Answered by Robert Dawson and Penny Nom.
Is one Infinity larger than another in the extended real number system? 2009-06-24
From Justin:
Hello there, I was wondering if one infinity is larger than another in the extended real number system (just like in the transfinite ordinals and cardinals with respect to infinite sets) or are all infinities the same size in the extended real number system? Thanks sooo much for answering my question! I greatly appreciate it!

All the Best,

Justin

Answered by Robert Dawson.
Two questions from math class 2009-06-18
From Con:
Hello,

My name is Con and my son is required to answer the following questions for his maths class.

He has attempted Q1 through trial and error and has found the answer to 72453. Is this correct?

He has attempted to draw the triangles described in Q2 in a number of ways and has found that BE can not equal ED and is dependent of angle BAC. Therefore, he claims that the triangle can not be drawn/practical. Is this correct or is there a slolution?

Q1.
Digits 2, 3, 4, 5 and 7 are each used once to compose a 5-digit number abcde such that 4 divides a 3-digit number abc, 5 divides a 3-digit number bcd and 3 divides a 3-digit number cde. Find the 5-digit number abcde.

Q2.
Let ABC be a triangle with AB=AC. D is a point on AC such that BC=BD. E is a point on AB such that BE = ED = AD. Find the size of the angle EAD. Con

Answered by Chris Fisher.
Divisibility 2009-06-17
From Sophia:
Hello
Please help my son with the solutions to the following:

a) Determine the remainder when 2^2009 + 1 is divided by 17;
b) Prove that 30^99 + 61^100 is divisible by 31;
c) It is known that numbers p and 8p^2+1 are primes. Find p.

Again, your assistance is greatly appreciated.
Thanks
Sophia

Answered by Robert Dawson.
The smallest 4 digit number 2009-06-17
From Kat:
What is the smallest 4 digit number? 0001 or 1000
Answered by Robert Dawson.
Triangular Numbers 2009-06-16
From Chinonyerem:
Question from Chinonyerem, a student:

Each of the numbers
1 = 1, 3 = 1+2, 6 = 1+2+3, 10 = 1+2+3+4 ,...
represents the number of dots that can be arranged evenly in an equilateral
triangle:
                            .
                  .        . .
        .         .       . . .             ...
.      . .      . . .    . . . .
This led the ancient Greeks to call a number TRIANGULAR if it is the
sum of consecutive integers, beginning with 1. Prove the following facts
concerning triangular numbers:
(a) A number is triangular if and only if it is of the form n(n+1)/2 for some n >= 1
(b) The integer n is a triangular number if and only if 8n+1 is a perfect square
(c) The sum of any two consecutive triangular numbers is a perfect square
(d) If n is a triangular number, then so are 9n+1, 25n+3, and 49n+6
Answered by Penny Nom.

You have the numbers 1 through 8, ... 2009-06-15
From Maryland:
You have the numbers 1 through 8, you can only use each number one time. Two of the numbers are multiplied together, two are subtracted, two more are subtracted and the last two are subtracted. All the answers are the same.
Can't get it, I have racked my brain trying to help.

Answered by Claude Tardif.
Number & Operation Puzzle 2009-06-12
From Francine:
How can I get the number 26 using 1,2,3,4,&5 and using addition, subtraction, multiplication and division. Each operator and digit must be used exactly once. My son presented me this question and I can't determine if it's even possible.
Answered by Janice Cotcher.
A number puzzle 2009-06-10
From Ann:
Hey,
    I have a math problem that I can't solve which just happens to be a "must-answer". The answer to the first problem is 25,452 and the second is 21,232. I know how to add but my teacher said that there is a pattern wherein you just have to look at the set of numbers and you already know the answer without really adding it first.  Here are the condition of the problem:
   1) the first, second and fourth numbers were provided by me. ( I have no idea how this works)
   2) the third and the fifth numbers were given by the teacher. ( She's the one who knows the technique)

# She says that there is a "pattern."
 Help me please as soon as you can. Thanks!!! Here it is:
         
         5454                                   1234
   +    3636                                   5678
         6363                                    4321
         2323                                    8765
         7676                                    1234
       _______                           _________
 
 
Please answer ASAP. Thank you!!!

Answered by Penny Nom.
A number puzzle 2009-05-17
From Gita:
Ok, we really need help for a homework problem due on Monday morning
I've been trying for 2 hours and can't figure it out---if I can't I'm not sure
how my son can....We've been given a puzzle with the following rules:
--The sum of each side must equal 15
--We can only use the numbers 1,2,3,4,5,6,7, 8
--A number can be used only once
(Hint: Think of all combinations of 3 numbers = 15)
--The puzzle looks like this:

_____ _______ ______

_____               ______

_____ _______ ______


Answered by Penny Nom.
A number sequence 2009-05-08
From alicia:
what is the next number 1,2,6,42,1806------? is it 1954?
Answered by Robert Dawson, Chris Fisher, Penny Nom and Claude Tardif.
15 pigs in 4 pens 2009-04-22
From Melissa:
A farmer has 15 pigs. He wants to put them all into 4 pens, have an odd number of pigs in each pen, and have no pigs left over. Also, he isn't thinking about having any for dinner, as they are his pets.
How does he do it?

Answered by Robert Dawson.
What is the original number? 2009-04-16
From Yueh:
A five digit number is written on a blackboard. Ron erases one of the digits and adds a newly constructed number to the original one. The result is 41751. What is the original number?
Answered by Claude Tardif.
A number puzzle 2009-04-15
From Julie:
Hi, please help me!!! I have a subtraction puzzle... 5 digits minus 4 digits = 33333 Only using numbers 1-9 once each. I can get 33323 but then thats it!!! Please how do I work it out? There must be a simple way?? Thankyou
Answered by Claude Tardif.
Four-digit numbers 2009-04-11
From jp:
How many four-digit numbers are there?
Answered by Penny Nom.
GCD (a + b, a - b). 2009-04-01
From Tomas:
Let a and b integer and relatively prime. Prove that:
GCD (a + b , a - b) = 1 or 2

Answered by Stephen La Rocque.
Choosing a 4 digit number at random 2009-03-24
From shabkhal:
If a four-digit number is chosen at random, what is the probability that the product of the digits is 12.
Answered by Harley Weston.
wo consecutive numbers with a product of 9506 2009-02-25
From sofie:
find two consecutive numbers with a product of 9506
Answered by Penny Nom.
More on cardinal numbers 2009-02-18
From Justin:
Hello again, I was just wondering that since the rules of Cantor's cardinal numbers in set theory do not apply to the infinity obtained by limits in calculus (ex. x->0, y=1/x=infinity), does that mean that this infinity is the largest quantity in both calculus and mathematics?

Justin

Answered by Robert Dawson.
In what base is 3x3= 10? 2009-02-14
From David:
In what base is 3x3= 10, 3x3=11, 3x3 = 12? is there a fast way to see this or do I have to create multiplication tables until I find the right one?
Answered by Penny Nom.
A four digit number 2009-02-12
From alex:
the number of tens plus the number of hundreds equals the number of thousands. the number of ones is half the number of tens and one more than the number of hundreds the hundreds digit is 2. what is this four digit number
Answered by Penny Nom.
The page numbers in a book 2009-02-10
From Nichole:
It takes 852 digits to number the pages of a book consecutively. How many pages are there?
Answered by Penny Nom.
The resulting number is the square of an integer 2009-01-30
From Randy:
A positive integer n is chosen. Then the product n ( n + 1 ) is computed and two digits are appended to the end of the product. The resulting number is the square of an integer.

Show that it is always possible to complete the process above or give a positive integer n for which the process cannot be completed.

Answered by Chris Fisher and Claude Tardif.
Missing three digits in a phone number 2009-01-30
From Elmer:
I have part of a phone number. the first 3 digits are 212, the last digit is four, and the three missing digits are all odd, how many numbers will I have to call to get the right number?

212- _ _ _ 4?

Answered by Stephen La Rocque and Penny Nom.
What is 301 million in word form? 2009-01-30
From chyamber:
What is 301 million in word form?
Answered by Penny Nom.
Three consecutive natural numbers 2009-01-26
From abby:
1.what is the smallest four-digit number that is the sum of the three consecutive natural numbers? what are the three consecutive natural numbers that give this sum?

2.what is the largest three digit number whose square root is a prime?

Answered by Penny Nom.
How many students were there 2009-01-25
From sooos:
After a math lecture in university, the students go to the cafeteria. EACH ONE of them buys a cheese sandwich and a cup of tea. They ALL pay $18.49. How many students were they?
Answered by Chris Fisher.
Multiplying negative numbers 2009-01-15
From stephanie:
hi, I'm currently working in a grade 8 class where the students are learning about integers. the students were asked to find examples of how integers are applied in real life. they were able to find aplications for: adding and subtracting both positive and negative integers. however, we could not find an example of where two negative integers would be either divided or multiplied together in real life. For example: (-2)(-4)= +8. please help.
Answered by Harley Weston.
A 4-digit number 2009-01-05
From Anya:
It is a 4-digit number

The tens are double the ones.

The thousands are double the tens.

The sum of the digits is 19.

What number is it?

Answered by Penny Nom.
What is a group of three numbers within a larger number? 2008-12-16
From kim:
My daughter brought home a question for math homework. What is a group of three numbers within a larger number?
Answered by Robert Dawson and Harley Weston.
Two numbers 2008-12-15
From HERB:
I am thinking of two whole numbers. When I add them , their sum is 123. When I subtract the lesser number from the greater number their difference is 45. What are my numbers?
Answered by Penny Nom.
Do all square numbers have an odd number or factors? 2008-12-11
From Amy:
Do all square numbers have an odd number or factors?
Answered by Victoria West.
Three one-digit numbers 2008-12-06
From JOANNA:
Could you please tell me what Adding Three One-Digit Numbers += 15 1-9 without using any number twice
Answered by Penny Nom.
Some number theoretic speculations 2008-12-04
From Andrew:
Another way of looking at the 'alternating parity polynomial', again based on Fermat's Little Theorem, is to substitute (a - b) for x in x^(p-1) - 1 as this is always divisible by any prime p. So, if one removes the "- 1", there is always a remainder of (1/p)! (I took up your challenge!)
.
.
.

Andrew

Answered by Chris Fisher and Victoria West.
The reciprocal of a mixed number 2008-11-08
From Trini:
Can the reciprocal of a mixed number be another mixed number? Explain, and give an example.
Answered by Penny Nom.
Compatible numbers 2008-11-07
From Erika:
When working with compatible numbers, when do you choose to round up or down? For example: In the problem: 373 divided by 4, one could round 373 to 400 and keep the 4 the same, and the answer would be 100; OR, one could round 373 down to 360 and keep the 4 the same, and the answer would be 90. When given the problem as an assignment, which way dictates how one would round, up or down?
Answered by Penny Nom.
A 4 digit number 2008-11-06
From Chris:
numbers 0 - 9 form a 4 digit number with no repetition.

It would round to 3000 if rounded to the nearest thousand

The digit in the tens place is twice the value of the digit in the thousands place.

It is an odd number.

It contains the largest digit.

It contains one digit less than 2.

The digit in the hundreds place is odd.

Can you find the 2 numbers?

Answered by Penny Nom.
Compatible numbers 2008-10-30
From Paul:
Use compatible numbers to estimate the quotient of 23.52 and 11.04
Answered by Penny Nom.
Is zero a number? 2008-10-21
From milan:
why people call 0 number?....0 is nothing. What is 1,2,3,4,5,6,7,8,9?than???this is not logic...thanks i dont know what else number means quantinty,but 0 means nothing.Thanks Mialn
Answered by Harley Weston.
A product of positive and negative numbers 2008-10-03
From kaylou:
Suppose you were given 13 numbers and asked to find their product. Seven of the numbers were positive, and the rest were negative. Would your product be positive or negative? Why?
Answered by Penny Nom.
Compatible numbers 2008-10-02
From jeff:
what exactly are compatible numbers, math problem is:
Estimate the total weight of 2 boxes that weigh 9.4 pounds and 62,6 pounds using rounding and compatible numbers. Which estimate is closer to the actual weight and why?

Answered by Harley Weston.
Compatible numbers 2008-09-25
From Katie:
I am having trouble remembering how to find compatible numbers used to find the estimate in the foloowing problem. Can you please help me solve it and explain it to me? My son is in 5th grade and I need help. Thank you. Here is the problem: 2,752 / 28 estimate: 90
Answered by Penny Nom.
Some number words 2008-09-25
From jodie:
My son would like to know the names of digits. When counting it starts ones, tens, hundred, thousand, millions, billions, trillion...he would like to know the order of what comes next and how far we have named them..
Answered by Penny Nom.
Two consecutive numbers 2008-09-21
From tomas:
what is the two consecutive numbers with a product of 4160?
Answered by Penny Nom.
Number of factors 2008-09-18
From Austin:
I am stuck on finding what type of number has exactly two factors and what type of number has an odd number of factors? I need some examples.
Answered by Penny Nom.
A mixed number 2008-09-04
From irena:
Need help writing this fraction as whole or mixed number 124/6
Answered by Penny Nom.
Aboriginal number groupings 2008-09-04
From Barbara:
Our current place value system is based on the notion of 10s. Did Aborginal people use a similar system or did they group numbers differently?
Answered by Harley Weston.
The product of the digits of a four digit number 2008-07-26
From Pete:
I am a student preparing for a competition and this was one of the prep problems: The product of the digits of a four digit number is 6x5x4x3x2x1. how many such numbers are there with this property?
Answered by Stephen La Rocque.
The number of digits in a really big exponent 2008-07-21
From Pete:
how would you find the number of digits in a really big exponent without a calculator?
Answered by Penny Nom.
???? x ? = ???? 2008-07-20
From waiyan:
????
x ?
????
using 1 2 3 4 5 6 7 8 9

Answered by Janice Cotcher.
Number of factors 2008-07-20
From pete:
how do you find the number of factors in a really big number without using a factoring program or listing the out?
Answered by Penny Nom.
Two numbers 2008-07-19
From Jerry:
while driving in my car early this morning, i 'discovered' something and want to ask if there is an equation that would fit it...
Here it goes..... take any two numbers (with the exception of two of the same numbers), multiply the first number by 2, add the difference of the original two numbers, and the outcome will be the same as if you added the original two numbers together...
example: 3+5=8... thus, 3+3+(5-3)=8...
if you chose to reverse the numbers then..... 5+3=8... thus, 5+5-(5-3)=8

i know, simple stuff.. but i just want someone to tell me what exactly this is.. is there a 'law' that describes this?

Answered by Victoria West.
equation? 2008-07-19
From Jerry:
while driving in my car early this morning, i 'discovered' something and want to ask if there is an equation that would fit it...
Here it goes..... take any two numbers (with the exception of two of the same numbers), multiply the first number by 2, add the difference of the original two numbers, and the outcome will be the same as if you added the original two numbers together...
example: 3+5=8... thus, 3+3+(5-3)=8...
if you chose to reverse the numbers then..... 5+3=8... thus, 5+5-(5-3)=8

i know, simple stuff.. but i just want someone to tell me what exactly this is.. is there a 'law' that describes this?

Answered by Victoria West.
Rational numbers 2008-07-16
From hinal:
list 12 rational numbers which lie between
a) -1 and 0
b) -3 and -3

Answered by Penny Nom.
Two numbers whose sum is 25 and whose product is 120 2008-07-10
From Michelle:
Solve the following problem in at least two different ways.
Find two numbers whose sum is 25 and whose product is 120.

I know how to solve this problem by creating two equations x+y=25 and xy=120 and then use substitution.

Are there any different ways of solving this problem?

Answered by Janice Cotcher and Penny Nom.
Probability and two digit numbers 2008-07-07
From Peter:
What is the probability that if you multiply two randomly selected two digit whole numbers, the result is greater that 100 and less than 200. Express your answer as a common fraction.
Answered by Janice Cotcher.
Choosing a number from 1 - 400 2008-07-04
From Peter:
The numbers 1-400, inclusive, are put into a hat. What is the probability that the first number chosen at random is a multiple of 4 or 17? Express your answer as a common fraction. How do you do this the question without a calculator?
Answered by Leeanne Boehm.
The sum of the digits of two-digit number is 9 2008-07-01
From Audrey:
the sum of the digits of two-digit number is 9. if the digits are reversed, the new number if 63 greater than the original number. find the number.
Answered by Penny Nom.
Some phone numbers 2008-06-30
From star:
What do you think the probability of finding the last 3 digits of a telephone number if the 1st digit begins with 7?
Answered by Janice Cotcher and Harley Weston.
A number that ends with 2 2008-06-23
From Jim:
A number ends in 2. If you were to place that digit at the beginning of the number, the resulting number will be twice the original. Find the smallest such number.
Answered by Stephen La Rocque.
Four digit combinations 2008-06-23
From star:
Can you you give me a list of all 4 digit combination starting with the number 7, for example 7123,7213.
Answered by Penny Nom.
How do you change bases? 2008-06-17
From Peter:
How do you change bases eg. 121 from base 3 to base 5?
Answered by Penny Nom.
Nine digit numbers 2008-05-21
From Alex:
List of Nine digit numbers, that can be divided by nine?
Answered by Janice Cotcher.
Converting a decimal to a fraction 2008-05-19
From judy:
please explain how to get from the decimal number .3437 to the fraction 11/32
Answered by Leeanne Boehm.
A problem with numbers 2008-05-18
From Peter:
When a certain number N is divided by d, the remainder is 7. If the original number N is multiplied by 5 and then divided by d, the remainder is 10, find d
Answered by Stephen La Rocque and Penny Nom.
Which is larger, 727464^512693 or 624610^518548? 2008-05-18
From Yahweh:
Lets say I have two very large numbers in the form of a^b: 727464^512693 and 624610^518548

I want to calculate which number is larger, but its unreasonable to type these numbers into a spreadsheet to get an answer right away. How could I determine the larger or the two numbers efficiently?

Answered by Stephen La Rocque.
Options 1 2 4 8 16 32 64 128 etc. 2008-05-09
From Abraham:
I have seen programs with options and I wish to create a similar program.

I have options 1 2 4 8 16 32 64 128 etc.
If I receive option 3 means option 1 and 2 are included. Option 20 means option 16 and 4 are included. that because of the sum of those numbers is the olny way to get the given number.

I want to know if is there a way
to know which number from options are included.
How I know:
1 is Included
2 is Included
4 is Included

Answered by Stephen La Rocque.
How can the distance between two points be irrational? 2008-05-07
From Dawn:
Yesterday my advanced 7th grade pre-algebra students were finding the distance between two points on a coordinate grid using the distance formula. A student asked me how come we or math I guess is able to represent a finite distance with an irrational number. I found this to be a very interesting question. I and the 7 other math teachers on my staff had discussed this and we have a variety of thing to say to the kid, but nothing that amounts to an actual answer. The students have studied what it means for a number to be irrational, and they cannot understand how that can "measure" a line that obviously stops. Got any suggestions?
Answered by Stephen La Rocque, Penny Nom, Claude Tardif and Victoria West.
(1-i)ln(1+i) 2008-05-02
From Kim:
I am stuck on the expansion of (1-i)ln(1+i)=(1-i)[ln(square root of 2)+i(3.14/4 = 2n3.14)]
Answered by Harley Weston.
A 6 digit number 2008-04-23
From rajiv:
write the smallest 6-digit number using three different digits with 6 in the ten thousands place and 1 in the hundreds place
Answered by Penny Nom.
Why can't a square be twice another square? 2008-04-20
From kim:
why can't a square number double another one
Answered by Penny Nom.
The sum of some 5 digit numbers 2008-04-05
From Tom:
There are 120 five-digit numbers that use all the digits 1 through 5 exactly once. What is the sum of these 120 numbers?
Answered by Penny Nom.
Compatible numbers 2008-04-04
From dawn:
Need help. Estimate the quotient. Tell what compatible numbers you used.

817divided by 4 =

Answered by Penny Nom.
Operation on the mixed numbers 2008-03-26
From bradley:
1) Perform the indicated operation on the mixed numbers below; write answer in simplest form:
8 1/3 – 2 1/4

2) Perform the indicated operations on the mixed numbers below; write answer in simplest form; note;: "•" denotes multiplication:
2 1/2 • 3 2/3 • 5 3

Answered by Stephen La Rocque.
A number puzzle 2008-03-22
From Cassidy:
1. Number has three digits
2. The second digit is an odd number
3. The first and third digits are even numbers
4. The number can be divided by five, six, and seven with a remainder of four.

How do you even start this type of problem?

Answered by Victoria West.
Is this an irrational number? 2008-03-22
From Tom:
Is it possible to build an irrational number with numbers that are just 1's and 2's where only finitely many 2's appear? If so, how do we build it and how do we know it is irrational?
Answered by Penny Nom.
The product of two numbers is 1,000,000,000 2008-03-14
From Henry:
What two numbers that don't in zero when multiplied equals 1,000,000,000 and same with 1,000,000,000,000,000,000.
Answered by Penny Nom.
Some three-digit numbers 2008-03-05
From Toni:
The following is a word pattern problem listed on my son's homework: write a set of three-digit numbers that contain one odd number and three even numbers. This confused the both of us because that would make his number a four digit number, right? please respond. Thanks.
Answered by Penny Nom.
One number is four less than five times another number 2008-03-05
From alice:
One number is four less than, five times another number. The total of the two numbers is 50. What are the two numbers?
Answered by Penny Nom.
Convert from base 6 to base 12 2008-03-02
From Jade:
I am trying to convert 13405 base 6 to a base 12 number. I am not allowed to go into a base 10 number. How will can I base 12 without doing anything with base 10? Thank you kindly.
Answered by Stephen La Rocque and Penny Nom.
What number has exactly 51 divisors? 2008-02-26
From Lisa:
What number has a Tau which equals 51? In other words, what number has exactly 51 divisors? It must be a square of some kind!
Answered by Penny Nom, Victoria West and Harley Weston.
Fact family 2008-02-21
From Anand:
Can you think of a fact family that has only two facts? Write the fact family
Answered by Penny Nom.
Decimal fraction to mixed number 2008-02-20
From cristian:
write 7.62 as a mixed number
Answered by Penny Nom.
I am a 4-digit number 2008-02-12
From Nickie:
I am a 4-digit number with no repeating digits. I am divisible by 5, my first two digits (left to right) make a number divisible by 3, and my first three digits make a number divisible by 4. Also, my digits have a sum of 19 and I have the digit 7 in the thousands place. Who am I?
Answered by Penny Nom.
7, 6, 5, 4, 3 and 2 digit numbers 2008-02-10
From Edward:
using the number 12345678 how many 7, 6,5,4,3 & 2 digit numbers can you make when not using any number twice in the same number
Answered by Penny Nom.
The sum of two numbers is 63 2008-02-07
From kirk:
The sum of two numbers is 63. The difference between the numbers is less than 10. Find all possible pairs.
Answered by Penny Nom.
Four digit numbers containing 85 2008-02-06
From Vonda:
How many four digit numbers contain the digit pattern 85 at least once and only once?
Answered by Stephen La Rocque.
A positive integer with 2007 digits 2008-02-06
From Tom:
Prove that there's a unique positive integer of 2007 digits lenght, divisible by 2^2007, and made up solely by the digits 2 and 5.
Answered by Penny Nom.
Multiplying decimal numbers 2008-02-05
From alwyn:
Why should when you Multiplying Decimal numbers is value becoming less and less? don't you think even decimal number is a quantity and in no chance when it multiplies its should become less or nil !!!

In fact all Multiplying and or adding the value will go up and only when you subtract and divide it should become less !!

Answered by Stephen La Rocque and Penny Nom.
Numbers and Operations 2008-02-02
From Charlotte:
On the TABE (Test for Adult Basic Education) test, one of the categories is number operation. I would like to know what consist of number operation.
Answered by Penny Nom.
A 3 digit number 2008-01-23
From Adrian:
Consider a 3-digit number,abc,where a is not equal to c.Switch it so that it becomes cba.Subtract the lesser from the greater to obtain rst.Switch this so that it is tsr.Prove that rst+tsr=1089
Answered by Stephen La Rocque.
Choosing 10 numbers from 40 numbers 2008-01-18
From Antwan:
I have 40 numbers........number 1-40. I want to know how many times i can chose 10 of those numbers without picking the same exact sequence twice if its even possible?
Answered by Stephen La Rocque.
The sum of two numbers is 36 2008-01-14
From May:
The sum of two numbers is 36.
Their product is 320.
What are the two numbers?

Answered by Penny Nom.
Find the lengths, and explain in a 'let statement' 2007-12-20
From Cordilla:
the length of the sides of a triangle are 3 consecutive whole numbers. The perimeter of the triangle is 102 meters. Find the lengths, and explain in a' let statement'.
Answered by Stephen La Rocque and Harley Weston.
Negative numbers 2007-12-14
From mannal:
what negative integers do we use in the real world? like -30 temperature?
Answered by Penny Nom.
A 3 digit number 2007-12-13
From Kim:
Find the number from the following clues:

A) is is a 3 digit even whole # divisible by 5
B} Each digit is different
C) Its hundreds digit is greater than its tens digit
D) It is less that 400, divisible by 3, and has only 1 odd digit.

Answered by Penny Nom.
Improper fractions and mixed numbers 2007-12-12
From Selah:
How do I change an improper fraction to a mixed number?
Answered by Penny Nom.
Imaginary roots 2007-12-09
From Josh:
What is the correlation between imaginary roots (of a quadratic or other polynomial equation) and the graph of the equation? As in, how can one represent imaginary solutions graphically (and why does that work)?
Answered by Harley Weston.
Three consecutive numbers 2007-12-08
From Aris:
The product of 3 consecutive numbers is 1716. Find their sum.
Answered by Stephen La Rocque.
Representing different bases 2007-12-05
From Sudhir:
What does a number subscripted to another one denote? (I know that a number superscripted to another denotes the power, but the subscript number is something I have never seen before.)
Answered by Stephen La Rocque.
How many marshmallows did I use? 2007-11-21
From gloria:
the # of marshmallows i use is an even # .it is 2 digit number more than 10 less than 20 the 10th digit half of one digit.how many marshmallows did I use?
Answered by Chris Langdon.
A 5-digit whole number 2007-11-19
From sunjoo:
a friend tells you to think of a 5-digit whole number. then do the following in order : multiply the number by 2. add 200 to the result. divide the result by 2. subtract 100 from the result. what is the greatest number you can get as an answer?
Answered by Stephen La Rocque.
Compatible numbers 2007-11-19
From Brendan:
Let's say you have 291/2 and its between 281/2=14 and 301/2=15. My question is where do the 14 and the 15 come from or in other words how do you get those?
Answered by Penny Nom.
x^2 + x^3 = n^2 2007-11-14
From Rapin:
x and n are the whole number and less than 100 , x^2 + x^3 = n^2, please help to solve this equation.
Answered by Penny Nom.
Whole numbers and improper fractions 2007-11-13
From Jennifer:
I would like to know how you convert whole numbers into improper fractions
Answered by Melanie Tyrer.
A ten digit number 2007-11-11
From jeff:
find the greatest ten-digit positive multiple of 12 using each digit once and only once
Answered by Penny Nom.
What number am I? 2007-11-07
From Orly:
what number am i? my tens digit is 2 more than my ones digit.I am less than 100.I am greater than 89. please explain to me how. thank you
Answered by Stephen La Rocque.
What is a rational number between -5.9 and -6? 2007-11-06
From cathy:
What is the rational number between -5.9 and -6?
Answered by Penny Nom.
A number puzzle 2007-10-31
From matthew:
please help.make 200.take 4 numbers from the numbers 1-9,place the numbers into a square containing 4 equal squares 1 number to each square.example 13
57
you now have 4 sets of numbers 13 ,57 ,reading across and 15 , 37. reading down i have to find 2 or more ways of making i have to find the numbers that when added together using this method make 200.you can only use each number once.

Answered by Penny Nom.
I am a prime number 2007-10-31
From sherry:
I am a prime number. I can not be shared fairly except by one group. My array can only be one row. array can only be one row. Can you please explain this out to me so I can help my child with it. Thanks
Answered by Penny Nom.
An array for 31 2007-10-31
From sherry:
How do you make an array with the number 31
Answered by Penny Nom.
Compatible numbers 2007-10-30
From Gary:
How do I estimate the quotient. and then tell what compatible numbers were used. 703/7
Answered by Penny Nom.
An irrational number 2007-10-29
From Clara:
Find an irrational number between 0.53 (with 53 as repeating) and 0.54 (with 54 repeating)

I changed each to 53/99 and 54/99 with 1/99 being the difference.

Please help me.

Answered by Stephen La Rocque, Penny Nom and Harley Weston.
Complex numbers 2007-10-27
From Dylan:
My problem is to prove:

|z|^2 = zz* Where z is the complex number x + iy and z* is it's complex conjugate x - iy.

If the absolute value of i is 1, then it looks like: |z|^2 = |x+y| |x+y| = x^2 + 2xy + y^2

And zz* = x^2 + y^2. for these to be equal, 2xy = 0. This seems wrong to me. What am I doing wrong?

Answered by Penny Nom.
The largest number you can add ... 2007-10-26
From Dryden:
What is meant by asking what is the greatest number you can add to that number without having to regroup in any place?
Answered by Penny Nom.
4 odd numbers that equal 21 2007-10-15
From gary:
4 odd numbers that equal 21
Answered by Penny Nom.
Show that there is no five-digit number which 2007-10-04
From Greg:
show that there is no five-digit number which uses each of the digits 1,2,3,4,5 such that the numbers formed by the first digit is divisible by 1, by the first two digits is divisible by 2 by the first three digits is divisible by 3, by the first four digits is divisible by 4, by the first five digits is divisible by 5
Answered by Stephen La Rocque, Claude Tradif and Victoria West.
Finding the last non-zero digits of large factorials 2007-10-04
From Mukesh:
i have to find last five non zero digits of integer which can be very large ( upto 10^12) . i can find last non zero digit of of any factorial. Now my problem is that i have to find last five non zero digit of factorial and also i want to general method for last K non zero digits of factorial n. For example 10!=3628800 so last non zero digit is 8 ,last two non zero digit is 88 .....and last five non zero digit is 36288.
Answered by Victoria West.
Equivalent mixed numbers 2007-10-02
From negra:
an average-sized person can burn about 6 1/2 calories a minute while ridinng a bike. Which of the following is equivalent to that amount?
a) 1 2/2

b) 5 6/2

c) 6 2/4

d)6 2/6

Answered by Stephen La Rocque and Penny Nom.
How many ten thousands makes one million? 2007-10-02
From Payton:
how many ten thousands makes one million?
Answered by Penny Nom.
I have a 6 digits 2007-09-28
From Kim:
I have a 6 digits. My hundred-thousands digit is 1 less that my ones digit, 8 more than my thousands digit, twice my tens digit, and 4 times my hundreds digit. My ten-thousands digit is 0. What number am I?
Answered by Stephen La Rocque.
Four digit odd numbers 2007-09-26
From sonia:
how many 4 digit odd numbers can be made using the digits 2, 5, 7, and 8
Answered by Penny Nom.
The largest 4-digit odd number 2007-09-21
From Courtney:
what is the largest 4-digit odd number you can make with 1 - 9. You can not repeat digits within a number
Answered by Harley Weston.
A fraction that cannot be simplified 2007-09-17
From Kevin:
Make a fraction that cannot be simplified and has 24 as its denominator?
Answered by Stephen La Rocque and Chris Langdon.
A 5 by 5 checkerboard 2007-09-17
From Darren:
Hi, I'm Darren and i have some questions to ask you about this problem: In a 5 by 5 checkerboard : how many 2 by 2 squares are there, what other sizes of squares do you need to count and how many of of each size of squares can you find; how many squares did you find in all
Answered by Victoria West.
Number sentences 2007-09-15
From jen:
list the addition number sentences that can be written using two sets of cards with the numbers 5 6 and 7. which number sentences have the same addends but show a different order?,,,,,,,,,,,,,,Ithis is for my son in 3rd grade and for the life of us we can't figure it out,,thanks for this elementary help
Answered by Harley Weston.
Three digit numbers where all digits are even 2007-09-14
From Cielo:
Hannah made a list of all of the three digit whole numbers where all three digit were even. How many three digit whole numbers were on Hannah's list?
Answered by Penny Nom.
A number story 2007-09-14
From Roxanne:
Make an array for the number sentence 4*8= 32

then they want me to write a number story for the number sentence.

Answered by Penny Nom.
Number pattern 8, 27, 64, ... 2007-09-12
From Valerie:
what is the number pattern 8,27,64,
Answered by Stephen La Rocque.
How many two digit numbers contain at least one 7? 2007-09-06
From Janet:
How many two digit numbers contain at least one number seven?
Answered by Penny Nom.
Compatible numbers 2007-09-05
From Beth:
Estimate using compatible numbers:

2 X 3978 =

102/25 =

Answered by Stephen la Rocque.
Compatible numbers 2007-09-01
From andrea:
Estimate using compatible numbers. 249 /64
Answered by Harley Weston.
A question about integers 2007-08-24
From Jerry:
Does there exist a positive integer such that when it is written in base 10 and its leftmost digit is crossed out, the new number is 56 times less than the original number?
Answered by Stephen La Rocque and Penny Nom.
How many three button codes are possible? 2007-08-23
From kim:
A home security device has ten buttons and is disarmed by pushing three buttons in sequence. How many three button codes are possible if each button can be pushed more than once in the code?
Answered by Stephen la Rocque.
Induction - divisibility 2007-08-04
From Jerry:
How would you prove that for any positive integer n, the value of the expression 3^(2n+2) - 8n -9 is divisible by 64.
Answered by Chris Fisher and Penny Nom.
Four digit numbers 2007-07-30
From Spence:
My son has a problem that he has been unable to figure out. I was asked to try, but I am confused on how to get the last two parts. Any help would be great.
Thanks
Spence

How many four-digit numbers can be formed under each condition?

A) The leading digit cannot be zero: 9(10)(10)(10)=9000

B) The leading digit cannot be zero and no repetition of digits allowed: 9(9)(8)(7)=4536

C) The leading difit cannot be zero and the number must be less than 5000: ??

D) The leading digit cannot be zero and the number must be even: ??

Answered by Penny Nom.
Multiplcation of two negative numbers 2007-07-26
From Brett:
Someone asked a question about multiplication and division of two negative numbers yielding a positive result here: http://mathcentral.uregina.ca/qq/database/QQ.09.99/butler1.html I was not fully happy with the explanation b/c I want to give me daughter a real-world example and I can't seem to find one.

The following illustrates why multiplying negative numbers has become difficult to explain:

2 X 2 = 4

----(-4)---(-2)---0---2---4
In this example we start with 2 and then want 2 more of them. When we move across the number line from 2 to our answer, which is four, we have moved only 2 units to the right.

-2 X -2 = 4

----(-4)---(-2)---0---2---4
In this example we start with -2 and then want -2 more of them. When we move across the number line from -2 to our answer, which is four, we have moved 6 units to the right.

How can the phenomenon of multiplying two negative numbers being more powerful than multiplying two positive numbers be explained? -Brett

Answered by Stephen La Rocque and Harley Weston.
(1 - i)^5 2007-07-24
From sofia:
Compute the given arithmetic expression and give the answer in the form a + bi where a,b element in R. 1. (1 - i)^5
Answered by Harley Weston.
A complex number in polar form 2007-07-23
From roland:
write the given complex number z in polar form lzl(p+qi) where lp + qil=1 for 3 - 4i.
Answered by Harley Weston.
If A is 1 and Z is 26 2007-06-29
From Sue:
If A is 1 and Z is 26 is there a word that when you multiply all the numbers instead of letters adds up to a million?
Answered by Chris Fisher and Harley Weston.
The number of blocks on the face of a pyramid 2007-06-27
From Heather:
I was wondering if there was an equation in order to calculate the number of blocks on the face of a pyramid. For example:

X
X X
X X X

Is there a specific formula in order to calculate that there are 6 blocks here? Thank You, Heather

Answered by Harley Weston.
Exponential form: x^y 2007-06-22
From Kishor:
whats the easy way to calculate X raised to Y where y is much greater than x.
Answered by Stephen La Rocque.
Simplifying complex denominators 2007-06-21
From Krys:
How do I simplify completely? ((4+i ) / (3+i )) - ((2-i ) / (5-i ))
Answered by Stephen La Rocque.
Compatible numbers 2007-06-15
From meryl:
what is a compatible number? ex.6,321/8
Answered by Penny Nom.
Repeating decimals and rational numbers 2007-05-31
From lil:
Why are repeating decimals considered rational numbers?
Answered by Penny Nom and Gabriel Potter.
What is the highest known numerical value? 2007-05-29
From Peg:
What is the highest known numerical value?
Answered by Gabriel Potter.
Positive rational numbers 2007-05-22
From Rebecca:
What is the definition of positive rationals/positive rational numbers?
Answered by Stephen La Rocque.
Fractions and mixed numbers 2007-05-08
From Amanda:
I am having truble with this problem: WRITE THE FRACTION 8/3 AS A MIXED NUMBER
Answered by Stephen La Rocque.
A problem involving squarefree integers 2007-05-07
From Andrew:
I was told that if x > y (integers); then x would never exactly = divide y^n (n integer > 1) if (x,y) =3D 1 ; or if x is "square = free". Is the latter true and why?
Answered by Stephen La Rocque and Penny Nom.
The sum of two or more consecutive integers 2007-04-26
From nana:
some numbers can be written as the sum of two or more consecutive integers(we consider positive integers only) other number (eg. 4) cannot be express in this way.
let us call a number which can be expressed in this way a SOALTCI( sum of at least two consecutive integers)
a). list the first few SOALTCI and conjecture the general formula.
b). Prove that any number of the given form(in your answer (a)) is a SOALTCI

Answered by Penny Nom.
Who am I? 2007-04-24
From Lisa:
I am the largest two digit number whose units digit divides its tens digit evenly. Who am I?
Answered by Stephen La Rocque.
24 four-digit numbers 2007-04-21
From Megon:
Of the 24 four-digit numbers formed from the digits 1,2,3,&4 is there an easier way to find their sum other than writing them all out and totaling them by hand?
Answered by Stephen La Rocque.
Three digit numbers 2007-04-19
From katelyn:
How many 3 digit numbers can be created if each number is a multiple of 10?
Answered by Penny Nom.
How many different ways can she purchase candy? 2007-04-18
From fathia:
Stephanie can buy lemon candies for 3 cents each and chocolate candies for 5 cents each. Stephanie has 64 cents to spend. How many different ways can she purchase candy if she wants to spend all her money?
Answered by Stephen La Rocque.
-12/(7 - i) 2007-04-18
From Diana:
Perform the operation. Write all answers in a + bi form.

-12
---------
7 - radical -1

Answered by Penny Nom.
A subtraction problem with negative numbers 2007-04-12
From Bridget:
my daughter has to change a subtraction problem with negative numbers in it to an addition problem with negative numbers and then solve the problem such as: -4-(-2)=?
Answered by Penny Nom.
The sum of two rational numbers 2007-04-04
From Fathia:
Hi, Could please help me with this question? Prove that if a and b are rational numbers then a + b is a rational number. Thank you,
Answered by Stephen La Rocque.
Algebra 2007-04-03
From Emily:
9,365 and 2,242 added to the difference between 255 and half of a number is 11,784. What is the number?
Answered by Haley Ess.
A four digit number 2007-03-28
From Paulo:
Hi , if i have to discover a keypad number of four digit , Which the closest combination of getting right?? i have this tips ---> i know the second and thirth number is odd . dont have the number 7 in any digit , Doesnt begin with 8 or 4 ... and dont have the 2 as a last number . Thanks !!
Answered by Stephen La Rocque.
Two consecutive odd numbers 2007-03-19
From Alicia:
The sum of two consecutive odd numbers is divisible by 4.

Can you make similar statements about the sum of three consecutive odd numbers and of four consecutive odd numbers? Generalize your findings.

I tried :
n= smallest odd number therefore the next = n, n+2 and the second =n+4 and the third = n+6 Now n+ n+2+n+4+n+6/4
Please help I am not sure of this I am just trying

Answered by Penny Nom.
The square of an odd number 2007-03-16
From Samantha:
Is the square of an odd number always odd?
Answered by Stephen La Rocque and Penny Nom.
Using complex numbers 2007-03-12
From Kara:
Do you use complex numbers in your job?
Answered by Stephen La Rocque and Penny Nom.
Seven digit account numbers 2007-03-09
From Miranda:
Account numbers for the Central Oil Company consist of seven digits. If the first digit cannot be 0, how many account numbers are possible? Thank you so much!!!
Answered by Penny Nom.
The tens digit of a really large number 2007-03-05
From Sai:
How can i find the tens digit of a really large number? i was gven 63^15 + 15^63 in a competitive exam.
Answered by Penny Nom.
Four digit numbers 2007-02-26
From Anton:
Hello! I would like to ask you how many combinations are possible using 4 digit number. # # # # (1...9).There is no zero,numbers don't repeat,second and forth are odd.Thanks
Answered by Penny Nom.
What are the two numbers? 2007-02-25
From Thomas:
Two numbers have a total of 32 and a difference of 14. What are the two numbers?
Answered by Stephen La Rocque.
A two-digit number is five times its units digit. 2007-02-21
From Pamela:
A two-digit number is five times its units digit. If the digits are reversed, the resulting number is 27 more than the original number. Find the original number.
Answered by Gabriel Potter and Penny Nom.
Exponential form of complex numbers 2007-02-12
From Austin:
When dealing with imaginary numbers in engineering, I am having trouble getting things into the exponential form. The equation is -1+i now I do know that re^(theta)i = r*cos(theta) + r*i*sin(theta). Just not quite understanding the order of operations. Thanks
Answered by Penny Nom.
An even positive integer cubed minus four times the number 2007-02-07
From Rachael:
I can't figure out the proof or the method to get the proof for this question: any even positive integer cubed minus four times the number is divisible by 48
Answered by Haley Ess and Penny Nom.
11^n +22^n = 55^n 2007-01-29
From Ankit:
11^n +22^n = 55^n

find the value of n?

Answered by Penny Nom.
Prime numbers 2007-01-16
From A student:
are the product of two prime numbers always prime?
Answered by Penny Nom.
Show that 3k+2 and 5k+3 are relatively prime 2007-01-15
From Andrea:
show that if k is a positive integer, then 3k+2 and 5k+3 are relatively prime
Answered by Penny Nom.
21 pigs and only 4 pig pens 2007-01-11
From Emma:
i was given this pig pen problem. if there are 21 pigs and only 4 pig pens how can you distribute the pigs into the pens and have an odd number of pigs in each pen. it doesn't seem possible to me. i need help!
Answered by Haley Ess.
A word problem 2007-01-05
From Harold:
the sum of one number plus four less than five times another is 26.
Answered by Penny Nom.
Phone number combinations 2007-01-04
From Efrain:
i want to know the different combinations for numbers 7 digits the number are 0-9 its basically the different phone number combinations
example: 555-6689 or 565-4896 i just want to know for many different combinations are there....

Answered by Penny Nom.
A word problem 2006-12-30
From Harold:
one number is less than 5 times another number. sum of the numers is 26
Answered by Penny Nom.
1, 3, 6, 10... 2006-12-15
From Lisa:
What is the nth term for the sequence, '1, 3, 6, 10...' and could you say how you got the anwser.
Answered by Stephen La Rocque and Penny Nom.
The absolute value of imaginary and complex numbers 2006-12-11
From Keith:
i don't get how to find the absolute value of imaginary and complex numbers here is an examples from the text book the answers are given but they don't show the work so i can follow along just show me the work please and explain how it is done

problem
3+4i

Answered by Stephen La Rocque and Penny Nom.
Four two-digit numbers 2006-12-02
From Mary:
I have four two-digit numbers written on my paper. The sum of these 4 numbers is less than 100.

True or False - each number is less than 25

If all the numbers on my paper are different, what is the largest (two-digit) number I could possibly have written.

Answered by Stephen La Rocque.
The sum of prime numbers 2006-11-28
From Rambabu:
I would like to know the sum of prime numbers below some X say 10000. Provide me with easy way to get this done
Answered by Penny Nom.
1X2X3X4+1=5^5 2006-11-23
From Liza:
1X2X3X4+1=5 square
2x3x4x5+1=11 square
What is the rule for this?

Answered by Stephen La Rocque and Penny Nom.
Factors 2006-11-22
From Jacqueline:
Why do the numbers 16,36,48,60,64,90 and 100 have an odd number of factors?
Answered by Stephen La Rocque and Penny Nom.
Pick any prime number greater than 3,square it ,then ... 2006-11-20
From Eliseo:
I was ask to pick any prime number greater than 3,square it ,then subtract 4, then divide the new result by 12 and record the remainder. He told me the remainder was 9. How could he be sure that the remainder was 9 without knowing which prime I picked?
Answered by Stephen La Rocque.
What is the highest number possible with 3 digits? 2006-11-16
From Liz:
What is the highest number possible with 3 digits? The teacher said the answer is not 999 and that the question did not say "3-digit number." My answer then is 9 to the 99th power (written mathematically of course) as this uses "3 digits" and is obviously a huge number. Is this correct?
Answered by Stephen La Rocque.
Number combinations 2006-11-09
From Bob:
how can I find out all the different combinations for the numbers 8168 example 8168, 8868, 1688,1886,6818 etc. how many are their.
Answered by Penny Nom.
The real numbers with decimal representations consisting of all 1s. 2006-10-29
From Ivessa:
Determine if the following set is countable or uncountable : the real numbers with decimal representations consisting of all 1s.
Answered by Steve La Rocque and Walter Whiteley.
An odd number of factors 2006-10-28
From Anthony:
What is the common name used for numbers that have an odd number of factors? This answer should be a two word answer only.
Answered by Stephen La Rocque.
Four digit combinations 2006-10-26
From Sasha:
CAN YOU LIST ALL OF THE POSSIBLE 4 DIGIT COMBINATION'S, THAT DO NOT START WITH 0?
Answered by Stephen La Rocque.
Three consecutive odd numbers 2006-10-21
From Pige:
The sum of three consecutive odd numbers is 387. Find the numbers.
Answered by Penny Nom.
Write the digits 1 to 9 on index cards, ... 2006-10-19
From Kim:
Write the digits 1 to 9 on index cards, arrange the cards to form three 3-digit numbers (using each card only ONCE). The third number is the sum of the first two numbers ** how many different ways can you do this?? **
Answered by Penny Nom.
The sum of two whole numbers is 72 2006-10-18
From Kelin:
The sum of two whole numbers is 72. Their differences is 48. What are the two numbers.
Answered by Stephen La Rocque and Penny Nom.
A specific 8 digit number in a 11 digit mobile phone number 2006-10-13
From James:
I need to know the chance of finding a specific 8 digit number in a 11 digit mobile phone number. the number must be 29571596 and the 11 digit phone number must end in this. All numbers 0-9 are used in a mobile number but the first 2 digits will always be 07. Is there a way to work this out??
Answered by Stephen La Rocque.
How many students are in the class? 2006-10-12
From Lillian:
There is one more boy than girls in the class, how many students in all?
Answered by Stephen La Rocque.
Four fours 2006-10-08
From Prabh:
Find out 10 BEDMAS problems using order of operation with only four 4's in the problem. The solution must be the digits 1-10. Example 44/44=1
Answered by Stephen La Rocque.
A number pattern 2006-10-02
From Tim:

The patterns that went in 2s, 4s and 5s we got, but we are stuck on the pattern that doubles. We can't come up with the rule or equation that solves it.

With the first number being the term number and the second being the term, this is the pattern:

1-1
2-2
3-4
4-8
5-16
6-32
7-64
100-128

What is the rule for this?


Answered by Stephen La Rocque and Penny Nom.
2 5 11 17 23 31 ? 2006-09-28
From Bernice:
How do you determine the next number in the following sequence:
2 5 11 17 23 31 ?

Answered by Penny Nom.
What number could it be? 2006-09-27
From Jonathan:
What number could it be?

It is between 12000 and 13000
Its hundreds, ten and ones digits are all odd
the sum of all its digits is 12.

Answered by Steve La Rocque and Claude Tardif.
A three-digit number multiplied by 7, then 11, then 13 2006-09-17
From Sherrie:
why is it true that a three-digit number multiplied by 7, then 11, then 13 will result in the three digits repeated twice?
Answered by Stephen La Rocque.
A remainder of 1 2006-09-13
From Juan:
What is the smallest counting number which when divided by any of the numbers 2,3,4,5,6,,7,8,9 and 10, leaves a remainder of 1?
Answered by Paul Betts and Claude Tardif.
How many different three-digit numbers can you make using ... 2006-09-12
From Beth:
How many different three-digit numbers can you make using the numbers 2, 5, 7, 8 using the digits any number of times?
Answered by Stephen La Rocque and Penny Nom.
How many thousands are in ten million. 2006-08-24
From Echoe:
How many thousands are in ten million.
Answered by Stephen La Rocque.
The last digit of a six-digit number is 2. 2006-07-09
From GG:
The last digit of a six-digit number is 2. If the 2 is moved to the start of the number, the new six-digit number is only a third of the original number. Find the original number.
Answered by Paul Betts, Chris Fisher, Penny Nom.
A number puzzle 2006-06-30
From Atitya:
The diagram given below consist of seven line segments and six circles,place the numbers 1 to 13 on either a line or a circle in such a way that the numbers on each segment is the difference of the two at its end points.
Answered by Stephen La Rocque and Penny Nom.
As close to 841 as possible 2006-06-21
From Alan:
using these numbers by either add subtract division multiply come up with the answer as close to 841
100 25 10 7 6 3

Answered by Paul Betts.
express the number 100 using the same figure 6x 2006-06-17
From A student:
express the number 100 using the same figure 6x
Answered by Stephen La Rocque.
13 57 91 11 31 51 ?? 2006-06-15
From Chastity:
im trying to determine the missing number in the sequence 13 57 91 11 31 51 ??
Answered by Claude Tardif, Steve La Rocque and Natasha Glydon.
how do i find i^22? 2006-06-12
From Sky:
how do i find i22?
Sky

Answered by Stephen La Rocque.
What is the sum of the first 100 whole numbers? 2006-05-31
From Jo:
what is the sum of the first 100 whole numbers?
Answered by Natasha Glydon, Paul Betts and Penny Nom.
Three questions 2006-05-17
From Diane:

Question:

I

What is the largest real number less than 10?
1- there is no largest real number less than 10 or
2- 9
or
3- 9.999...

II

Given that not (a and b) implies c.What does
(not c) imply ?
1-a and b
2- (not a and b) or (a and not b)
3- (not a and b) or (a and not b)
4- a or b

III

What are the x and y intercepts of the equation
2x + 3y -6 =0 ?
1- x=3 and y=2
2- x=6 and y=-2
3- -3x + 2y =0


Answered by Paul Betts.
What is the radius of the sphere? 2006-05-17
From Jimmy:
The surface area and volume of a sphere are both equal to the product of 4-digit whole numbers and pi. What is the radius of the sphere?
Answered by Stephen La Rocque.
A number theory problem 2006-05-05
From DeHayward:
Find the 6-digit number in which the first digit is one less than the second, the third is half of the second, the fourth is 3 times the third, and the last 2 digits are the sum of the fourth and fifth. The sum of all the digits is 24.
Answered by Paul Betts.
The sum of a three digit number and its three individual digits is 429 2006-04-08
From Megan:
Gill has recently moved to a new house, which has a three- digit number. the sum of this number and its three individual digits is 429. What is the product of the three digits which make up the house number?
Answered by Chris Fisher.
Four digit numbers 2006-03-25
From Kenvin:
i was wondering how many possibilities is there for a four digit number with the same first two numbers and the last two is not one of the first with numbers 1-9
Answered by Stephen La Rocque.
Five digit numbers 2006-03-25
From Tony:
Can someone tell me how many unique five digit numbers can be generated using 0-9?
Answered by Stephen La Rocque and Penny Nom.
given that p is a prime and p|a^n, prove that p^n|a^n 2006-03-24
From Janna:
given that p is a prime and p|an, prove that pn|an
Answered by Stephen La Rocque.
Back to the nines 2006-03-15
From Victoria:

Can you answer this problem, does an answer exist?

  • Get a set of numbers 1-9 !
  • Using the whole set of nine number tiles (digits 1-9), try to arrange them to make three 3-digit numbers so that the sum of the first two is the third.
Can this be done without carrying over? If not can it be done without carrying over into the hundreds column?

Answered by Claude Tardif.
A nine digit number 2006-03-06
From Ryan:
What is the total number of possible combinations of a nine digit number (ie., social security number) including repeating numbers?
Answered by Stephen La Rocque and Penny Nom.
The meaning of numbers 2006-03-03
From John:
I'm having a philosophical debate on the meaning of numbers, equations, or the world of math in general. Would it be possible if you could help me by giving me a resource that talks about it or if you have your own opinion I would be most grateful.
Answered by Harley weston.
A triangle of 50p pieces 2006-02-22
From Stuart:

Ok, so i am collecting 50p pieces and arranging them on my desk in the shape of a triangle.
eg
50p
50p 50p
50p 50p 50p
50p 50p 50p 50p

I want to work out how much money I'm saving just by knowing how many rows of coins there are. If i can work out how many coins there are just by knowing how many rows I have I can just divide by 2 to find out the amount in dollars.


Answered by Penny Nom.
sinh(i/2) 2006-02-09
From Louis:
How can you set up an equation to find sinh(i/2)
Answered by Penny Nom.
An array problem 2006-01-31
From Cynthia:

Write a number sentence for this array........

It's a rectangle box 3 down and 4 wide.


Answered by Penny Nom.
Write the array for 3 2006-01-29
From Jamie:
My son is a 5th grader and came home with a math question that I can't answer. The question stated - "Write the array for 3" I don't understand how this can be done with just one number.
Answered by Penny Nom.
GCF, LCM, primes and the ladder method 2006-01-07
From Linda:
How would I teach both finding the GCF and LCM with prime factors...I recall the ladder method vaguely.???
Answered by Penny Nom.
The average of 4 numbers is 5. 2006-01-02
From Sheri:
The average of 4 numbers is 5. I have 3 of the numbers already- 5,8,4,? How do i find out what the last number to give me the average of 5?
Answered by Penny Nom.
Tables with perfect squares 2005-11-30
From Craig:
A table consists of eleven columns. Reading across the first row of the table we find the numbers 1991, 1992, 1993,..., 2000, 2001. In the other rows, each entry in the table is 13 greater than the entry above it, and the table continues indefinitely. If a vertical column is chosen at random, then the probability of that column containing a perfect square is:
Answered by Claude Tardif.
The square root of i 2005-11-30
From Kevin:

If the square root of -1 is i, what is the square root of i?

How can you find the log of a negative number?

What is the log of -1?


Answered by Claude Tardif.
100, 000 or 100 000 2005-11-21
From Jacqui:
I am a parent of a grade 6 student and need to confirm that commas are no longer used when writing numbers (the Canadian method). For example, how do you write one hundred thousand? 100, 000 or 100 000
Answered by Diane Hanson and Harley Weston.
Add one 1 at the beginning and at the end of a number 2005-11-13
From Samuel:
If we add one 1 at the beginning and at the end of a number, this number is increased by 14789. What is the sum of the digits of the number?
Answered by Penny Nom.
5 digit numbers that include 5 and exclude 8 2005-11-04
From Paul:
figure out how many 5 digit numbers are there that include the digit 5 and exclude the digit 8. show me how to do get to the answer. (23816)
Answered by Paul Betts.
A sum of 75 and a difference of 13 2005-10-30
From Vicki:
Find two numbers that have a difference of 13 AND A SUM OF 75? We need to learn about the formula or procedure.
Answered by Penny Nom.
Two 3-digit numbers 2005-10-27
From Melissa:
I can't figure this out, The question is

Find the greatest possible difference of two 3-digit numbers that use the digits 3, 5, 7 and 9 only once in each number.

The teacher says the answer is 62 but I can't figure out how she did it. Can you help.

Answered by Claude Targif and Penny Nom.
What number am I? 2005-10-27
From Samantha:
First 8 is added to me, then I am multiplied by 6. Then 40 is subtracted from me. Finally, I am divided by 10. The result is 11. What number am I?
Answered by Penny Nom.
Improper fractions 2005-10-21
From Joe:
Is a whole number an improper fraction. For example, is 4 an improper fraction? Is 4 an improper fraction? Is 4 the simplest form for 20/5? Are 4 and 20/5 equivalent fractions?
Answered by Penny Nom.
Divisibility by each of the first ten counting numbers 2005-10-17
From Simon:
determine smallest positive integer that is divisible by each of the first ten counting numbers
Answered by Penny Nom.
What is the number? 2005-10-10
From Jess:
One eighth of the square root of 7 less than a number is 2. What is the number?
Answered by Penny Nom.
Whole numbers 2005-10-10
From Kirk:
Can the sum of two or more whole numbers be less than any of those numbers?
Answered by Penny Nom.
Four digit numbers 2005-10-09
From David:
I had a discussion with my father about how many four digit numbers you can make with 0-9. He was saying that there is 12,000 and that seems quite high for me. But can you tell me how many there are, and what exactly are they?
Answered by Penny Nom.
Three digit numbers 2005-10-04
From Jordan:
i need to know how many 3 digit numbers there are
Answered by Penny Nom.
Use the numbers and symbols only once to come up with the answer 2005-09-19
From Kaitlyn:

Question: I have math problem that no one in my class was able to solve but the teacher said it does have an answer. We need to use the numbers and symbols only once to come up with the answer:

36 18 12 6 + - ( )

______________________________ = 41


Answered by Claude Tardif.
What is my number? 2005-09-16
From Bob:
I have 6 digits. 100,000th digit is one less than my ones digit, eight greater than my 1,000's digit, twice my tens digit and four times my 100s digit. My 10,000s digit is 0 What is my number?
Answered by Penny Nom.
A two digit prime 2005-09-12
From A student:
The units digit of a two-digit prime number is not greater than the tens digit by 3.If the positions of the two digits are interchanged, then the new number formed is greater than the original number by more than 18. Find the value of the original prime number.
Answered by Penny Nom.
n^2+n-1 has no divisors ending with 3 or 7 2005-09-08
From Arne:
at least it seems like for any integers n and k,
10k+3 and 10k+7 do not divide n²+n-1
I tested this for every n from 0 to 3200 (which means same for the numbers from -3201 to -1)
could this be true, or is it just coincidence, or am I just totally wrong?

Answered by Richard McIntosh.
Adding positive and negative numbers 2005-09-03
From Billy:
46 + -24 =
-38 + -22 =
19 - 20 =

Answered by Penny Nom.
1,4,9,1,6,2,5,3,6,4,9,6,4,8,1 2005-08-30
From Liz:
Find the next four numbers to the sequence 1,4,9,1,6,2,5,3,6,4,9,6,4,8,1,___,___,___,___.
Answered by Penny Nom.
1_2_3_4_5_6_7_8_9 = 1: Fill in the blanks 2005-08-26
From James:
Try to replace the blanks below with + or - to make the statement correct
1_2_3_4_5_6_7_8_9 = 1

Answered by Penny Nom.
Their sum equals their product 2005-08-21
From Claudio:
Find a thousand natural numbers such that their sum equals their product.
Answered by Penny Nom.
Can one place 15 integers around a circle... 2005-08-20
From Hernan:
Can one place 15 integers around a circle so that the sum of every 4 consecutive numbers is equal either to 1 or 3?
Answered by Claude Tardif.
Which number is greater? 2005-08-18
From Dante:
Which number is greater: 888....88 X 333...33 or 444.......44 x 666........67 (each of the numbers has 1989 digits)?
Answered by Claude Tardif.
Converting rational numbers 2005-08-07
From Joe:

I am helping my son with his converting rational
numbers in the form of A/B where A and B are integers and
B not equal to zero

I think I am doing this right but I am not sure so
below are the problems and our answers are beside
them, please let me know if these answerers are correct


Answered by Penny Nom.
A set of numbers whose sum is 1 2005-07-21
From Santos:
Does there exist a set of numbers whose sum is 1, and the sum of whose squares is less than 0,01?
Answered by Paul Betts.
Seven digit numbers 2005-06-23
From Ky:
How many number combinations are possible for a 7-digit number? How many people in a population can exist without having to exceed the seven digit combinations and having to revert to a 8 or 9 digit number?
Answered by Penny Nom.
Primes 2005-06-04
From Fede:
Prove that there is not a number prime p, for which the numbers p+5 and p+10 are prime too.
Answered by Penny Nom.
The numbers p and 8p^2 +1 are prime. 2005-05-30
From Antonio:
The numbers p and 8p2 +1 are prime. Prove that the number 8p2+2p+1 is also a prime number.
Answered by Claude Tardif and Penny Nom.
5+5+5=550 2005-05-30
From Bill:

5+5+5=550

This is a math equation that is not true. How can it be made to be true by using only one line?
Answered by Leeanne Boehm.

Which is larger, 31^11 or 17^14? 2005-05-29
From Linda:
Which number is greater 3111 or 1714 (without using calculator).
Answered by Chris Fisher and Paul Betts.
50^99 and 99! 2005-05-23
From Romel:
Which number is greater, 5099 or 99!
Answered by Claude Tardif.
Which is smaller 2^175 or 5^75? 2005-05-18
From Sarah:
Which is smaller 2175 or 575? without using a calculator.
Answered by Claude Tardif.
Divisibility of a^2 + b^2 2005-05-16
From Ampa:
given natural numbers a and b such that a2+b2 is divisible by 21, prove that the same sum of squares is also divisible by 441.
Answered by Penny Nom.
A pattern of numbers 2005-04-21
From Claire:
We need to be able to discover the nth term in a pattern of numbers and explain how we did it. We have arrived at the formula 1+2+3+...n=n(n+1)/2 for trios where a trio series is;

3 has the following trio (1,1,1)
4 has the following trios (2,1,1), (1,2,1), (1,1,2)
5 has the following trios (3,1,1), (1,3,1) (1,1,3) (1,2,2) (2,2,1) (2,1,2)

So the series is as follows

Answered by Penny Nom.
Some four digit numbers 2005-04-16
From Lori:
can u give me every possible combination of four numbers starting with the number three
Answered by Penny Nom.
Prime numbers 2005-04-11
From Jesule:
Given that p, p+10 and p+14 are prime numbers, find p.
Answered by Claude Tardif and Chris Fisher.
Rational functions 2005-04-05
From Nicole:
My name is Nicole and I am a teacher at Weyburn Comprehensive School. I am currently teaching both Math B30 and Calculus 30 at the school and I have a question about rational functions. I know that if a rational function (by definition) has common factors in the numerator and the denominator then it is not a rational function (math b30) however in calculus this common factor creates a hole in our graph. Can you explain to me why a common factor or constant does not give us a rational function?
Answered by Penny Nom and Leeanne Boehm.
When dividing a 3-digit number by a 1-digit number... 2005-01-22
From A student:
When dividing a 3-digit number by a 1-digit number, for what divisors can you get a remainder of 8?
Answered by Penny Nom.
Numbers that John likes 2005-01-16
From Garrett:
John likes 400 but not 300; he likes 100 but not 99; he likes 3600 but not 3700. Which does he like?

900
1000
1100
1200

Answered by Penny Nom.
Six digit numbers with at least one 7 2004-12-27
From Behzad:
How many six-digit numbers contain at least one 7 in their decimal expansion?
Answered by Penny Nom.
Making the number 99999 2004-12-22
From Lisa:
Make as many equations as possible to make the number 99999 using all of the numbers 0-9 but only once per equation. example 01234 + 98765 = 99999 she needs to make 150+ equations.
Answered by Paul Betts.
Some numbers less than 100 2004-09-17
From Paul:
for how many numbers less than 100 is the digit in the ones place half the digit in the tens place?
Answered by Penny Nom.
Number combinations 2004-09-10
From Angela:
I need the number of possible number combinations, both 3 and 4 digit, from 0-59

I know that 0-9 has 10000 combinations for 4 digits I also know that 0-9 has 1000 combinations for 3 digits

Perhaps you could explain an easier way to figure this out rather than writing all these numbers down??

Answered by Penny Nom.
Half way between 2004-09-08
From Ben:
Find the number halfway between the number shown

751,843

Answered by Penny Nom.
A big number 2004-08-26
From Mark:
If you have $999,999,999,999,999,999,999,999,999,999.99 and you find a pennie how much is that and can you give me the name of it.
Answered by Penny Nom.
A big number 2004-08-08
From Carter:
Can you tell me what number this is?
10,000,000,000,000,000,000,000,000,000,000,000,000

Where can I find the names for big numbers?

Answered by Penny Nom.
an integer with three factors 2004-08-03
From A student:
What is the probability that a randomly chosen 3 digit number has exactly 3 factors
Answered by Penny Nom.
Four digits 2004-07-10
From Rob:
I am always interested to know how the for digit works. I mean, you have correctly put it that the total numbers obtain from 0000 thru 9999 are 10,000. My question is, is there any simple formula to turn this simple 4 figures of say: 1234.
Answered by Penny Nom.
Factoring integers 2004-07-02
From A student:
After looking at all the info I could get about NFS, I still have some questions that are unsolved:

First of all: If someone found an algorithm that has a worst case running time of N*Log(N) to factor an integer n into his divisors, would it be quicker or slower then the number field sieve algorithm?

secondly, what exactly is the time complexity of the Number Field Sieve algorithm, if I would factor an integer n?

Answered by Claude Tardif.
Aunt Lucy's letter 2004-06-27
From Olivia:
"Dear bob, now that i am getting on (70 today). i want to give you some of my money, i shall give you a sum each year, starting now. you can choose which of the following schemes you would like me to use.

a) £100 now, £90 next year,£80 the yaer after and so on.
b) £10 now, £20 next year, £ 30 the next year and so on.
c) £10 now, 1.5 times as much next year, 1.5 times as much the year after that and so on.
d)£1 now, £2 next year, £ 4 the year after, £8 the year after and so on.

of course these schemes will only operate while i am alive. i look forward to hearing which scheme you choose and why! Best wishes, Aunt Lucy."

Answered by Penny Nom and Claude Tardif.
Pick a number greater than 1 2004-06-25
From A student:
I understand that when you pick a number greater than 1 and less than 10; multiply it by 7 and add 23, then add the digits of that number until you get a one digit number. Then multiply that number by 9, add the digits of that number until you get a one digit number, subtract 3 from that number and divide the difference by 3; that this process will always give you the result of 2. Does this have a name or theory for it as to why the answer will always be 2?
Answered by Penny Nom.
The Number of the Beast 2004-06-13
From A Heinlein reader:
Most people who have an interest in mathmatics are familiar with Robert Heinlein's novel "The Number of the Beast", where 666, or ((6 to the 6th power) to the 6th power) equals 1.0315 times 10 to the 28th power, which in the novel is the number of parallel universes in the cosmos. My question is what would the number of parallel universes be if you grouped 666 the other way: (6 to the power of (6 to 6th power)). I have tried this on my calculator, but it won"t register that high.
Answered by Penny Nom.
Jack's social security number 2004-06-08
From A student:
Jack's social security number contains each of the nonzero digits exactly once. By examining the digits from left to right, he also found that 1 divides the first digit evenly, 2 divides the sum of the first two digirs evenly, 3 divides the sum of the first three digits evenly, and so on, until 9 divides the sum of all the digits evenly. What is Jack's social security number?
Answered by Penny Nom.
Divisors of 2n+1 and 3n+9 2004-06-02
From Chris:
Amy selected a positive integer N and wrote numbers 2xN+1 and 3xN+9 on a whiteboard. Boris looked at the two numbers on the whiteboard and wrote in his notebook a positive integer that divides each of them. Find all possible numbers that Boris could have written in his notebook.
Answered by Chris Fisher and Penny Nom.
Numbers around a circle 2004-03-28
From Rebecca:
my maths question is use the numbers 1,2,3,4,5,6 and 7 place each number in a circle so each line adds up to 12. There are seven circles, six on the outside and one in the middle. Each number lines up with the middle number and the outside numbers line up with the one directly across from it as if a line was going through the middle number circle.
Answered by Penny Nom.
z^2 = 3 - 4i 2004-03-26
From John:
Solve: Z^2 = 3 - 4i
Answered by Harley Weston.
Large exponents 2004-03-26
From John:
I have encountered problem with the lack of memory every calculator seems to have. No calculator, on or off the computer, I've found has the amount of memory or writing space to calculate the sums I want to solve. The sums are in great importance for my continued progress. The sums are following:

16777216^1310270, 16777215^995328, 16777215^786432 and 16777215^480000

Answered by Claude Tardif.
Even-numbered roots 2004-03-01
From A student:
I understand that the square root is defined to be positive only. Does the same rule apply to other even-numbered roots, such as the 4th root, or the 6th root, etc.?
Answered by Harley Weston.
Some number problems 2003-12-24
From Shruthi:
1. Write 31 using 3 five times
2. Write 100 using all the 10 digits just once
.
.
.

Answered by Penny Nom.
400, 100 and 2500 2003-12-21
From A student:
A person likes 400 but dislikes 300
He also likes l00 but dislikes 99
He also likes 2500 but dislikes 2400

Which of the following does John like
900, 1000, 1100 or 2400


Answered by Penny Nom.
Four digit numbers 2003-11-20
From Rob:
Hello, I sure hope you can answer this question. I got into a discussion at work about how many possible combinations there are for a four digit number using the numbers 0 through 9. This was in reference to a discussion about the lottery. I said there was only 10,000 possible combinations. My friend said that there was many times more, but he could not remember how to figure it out. The answer and the formula would be greatly appreciated.
Answered by Penny Nom.
What is a number that has exactly 13 factors? 2003-11-09
From Mary:
What is a number that has exactly 13 factors?
Answered by Penny Nom.
The product of the page numbers 2003-11-07
From Morgan and Mom:
You open a book and the product of the page numbers is 12,656. What are your page numbers?
Answered by Penny Nom.
Which one has the most factors? 2003-10-31
From Kristi:
Of all the whole numbers less than or equal to 5000, which one has the most factors?
Answered by Claude Tardif.
Pairs of prime numbers 2003-10-13
From Nikolas:
Use pairs of prime numbers to find all the numbers less than 50 that have only two prime factors. Make an organized list.
Answered by Penny Nom.
Primes 2003-09-23
From Julie:

#1. Suppose you are given a number (n) and told that 1 and the number (n) divide into (n). Does that mean (n) is prime?

#2. Suppose that p is a prime number greater than or equal to 3. Show that p+1 cannot be a prime number.


Answered by Penny Nom.
The Sieve of Eratosthenes 2003-09-18
From Lynn:
My daughter has been asked to find all the prime numbers by using the Sieve of Eratosthenes. I have no understanding what this means.
Answered by Penny Nom and Claude Tardif.
How high does the sequence of numbers go 2003-09-07
From Shayna:
My quetion is "How high to the sequence of numbers go"?
e.g. one, ten, hundred, thousand, million, billion, trillion, and from there is my queston.

Answered by Penny Nom.
Two row arrays 2003-09-03
From Madalyn:
My question is, what are #'s that can not be arranged in two row arrays called?
Answered by Penny Nom.
X.9999... and X+1 2003-08-23
From David:
I have read your answers to the questions on rational numbers, esp. 6.9999... = ? and still have a question: The simple algebraic stunt of converting repeating decimals to rational numbers seems to work for all numbers except X.999999.... where X is any integer. The fact that the method yields the integer X+1 in each case seems to violate the completeness axiom of the real numbers, namely that there is no space on the number line which does not have an number and conversely that every geometric point on the number line is associated with a unique real number. In the case of 3.999... for example, it seems that both the number 4 and the number 3.9999.... occupy the same point on the number line. How is this possible???
Answered by Penny Nom.
39 consecutive natural numbers 2003-08-19
From A student:
Prove that among any 39 consecutive natural numbers it is always possible to find one whose sum of digits is divisible by 11.
Answered by Penny Nom.
nine digit numbers 2003-07-23
From John:
With a nine digit number, with each of the nine digits having a possibility of 10 different numbers then what is the total number of possible mathematical variations in the nine digit number. i.e. Social Security numbers have nine digits and if each of the nine digits have a possibility of being any one of ten numbers, i.e. 0,1,2,3,4,5,6,7,8,9. Then what is the formula to calculate the maximum possible number of variations in this nine digit number and what is the mathematical maximum possible number of variations of this nine digit number?
Answered by Penny Nom.
Numbers John likes 2003-06-20
From Steve:
John likes 400 but not 300; he likes 100 but not 99; he likes 2500 but not 2400.

Which does he like?
900
1000
1100
1200

Answered by Penny Nom.
The sum of two numbers is 5 and their difference is 2003-06-16
From Akhil:
The sum of two numbers is 5 and their difference is 11. What is the product of the two numbers?
Answered by Penny Nom.
Real numbers 2003-05-09
From Sirena:
what is a "real" number
Answered by Penny Nom.
Harmonic numbers 2003-03-19
From Becky:
Harmonic numbers are Hn = 1 + ? + 1/3 + . . . + 1/n

Use induction to prove the following theorem:
For all natural numbers n, H1 + H2 + . . . + Hn = (1+n)Hn - n

Answered by Penny Nom.
1 followed by a million zeros 2003-03-19
From David:
What do you call the number represented by the numeral '1' followed by one million zeros?
Answered by Penny Nom.
1575 2003-03-13
From Sharyn:
find 6 integers that when multiplied with each other equal +1575 and when the same integers are added together equal zero. there are more positives than negatives and one of the numbers is a double digit between 10 and 20
Answered by Penny Nom.
Harmonic numbers 2003-03-12
From Becky:
What can you tell me about the limit of harmonic numbers as it reaches infinity?
Answered by Penny Nom.
Three digit number 2003-03-10
From Grace:
What three digit positive integer is exactly 32 times the sum of its digits?
Answered by Claude Tardif.
A number line 2003-02-27
From Shery:

My seventh grader problem of the month

0__________1______________5____>

This is a arrow, the number should be below

A.Mrs Decker created an arrow representing a number line shown above. She wanted to find points and label them with a heart (G) for Valentine's Day so that the fraction 5/g is less than 1. (be sure to mark the G and not the fraction 5/g). She pondered, "Are there any other locations for G?" Is so help her description the location of all these points. If not why not?


Answered by Penny Nom.
Hundreds, thousands, millions,... 2003-02-19
From Karissa and Jasmeen:
hundreds, thousands, millions, - can you help us with the rest of this sequence - we are trying to find the largest number
Answered by Penny Nom.
6 digit numbers from 0,0,2,2,4,4 2003-01-23
From Amanda:
How many 6 digit numbers can you make from the numbers 0,0,2,2,4,4, giving that 0 cannot come first. The number has to contain 2 twos, 2 fours, and 2 zeros.
Answered by Penny Nom and Claude Tardif.
What is larger than infinity? 2003-01-12
From Dana:
What is larger than infinity?
Answered by Claude Tardif and Harley Weston.
ALL of the possible 3 digit combinations of 0 - 9 2002-12-09
From Melissa:
I need to find out ALL of the possible 3 digit combinations of 0 - 9. Eg: 000,001,002,003. This is going to take me VERY LONG time. Any suggestions.
Answered by Penny Nom.
65,125 2002-12-04
From Brittany:
I am a three-digit number.When you reverse my digits,a larger number is formed.the product of me and the new number is 65,125.What number am I.
Answered by Chris Fisher and Claude Tardif.
Making 7 2002-11-26
From Bill:
At one time I had the answer to this math equation but I have lost it and can't seem to regain it. I may be going at it wrong but as I recall there were the numbers 1 through 4, and the std operators, +, -, /, *, and parentheses. The object was to make the four numbers with the operators equal to 7. Each number and operator may be used only once. My 9 year old loves these quizzes but I can't give it to her if I can't provide the correct answer.
Answered by Claude Tardif.
4 x abcd = dcba 2002-11-20
From A student:
Say you have a four digit number (e.g. abcd) and you multiply the number by 4. The answer you get will be the reverse order of the number you started with (dcba). What is the number? The four digits (a,b,c,d) cannot be the same number or cannot be repeated.
Answered by Penny Nom and Claude Tardif.
abc,abc 2002-11-20
From Pam:
Prove or disprove that "every number of the form abc,abc (where a, b, and c represent digits) is divisible by 7,11, and 13"
Answered by Penny Nom.
Why is 5 divided by 1/7 greater than 5 divided by 2/3? 2002-11-19
From Elizabeth:
  1. Without performing the division, explain why 5 divided by 1/7 is a number greater than 5 divided by 2/3.
  2. Is the least common multiple of two prime numbers always their product? Why or why not?

Answered by Diane Hanson and Penny Nom.
The product of two consecutive numbers is 41 more than their sum 2002-11-15
From A mother:
THE PRODUCT OF TWO CONSECUTIVE NUMBERS IS 41 MORE THAN THE SUM OF THE NUMBERS. WHAT ARE THE NUMBERS?
Answered by Penny Nom.
Prime numbers 2002-11-08
From Lonna:
Is 2 a prime number or not??
Why is 1 not considered a prime number?

Answered by Penny Nom.
Relatively prime 2002-10-04
From Natasha:
I really need help with this middle level math question. My little brother is asking me and I have no clue what the answer is. Explain what it means when two numbers are negatively prime. (?)
Answered by Kathy Nolan and Penny Nom.
Number play 2002-09-24
From Julie:
Organize these numbers and the operations so that each equation is correct. Use each coice only once per equation.

36 18 12 6 - + ( ) /

_________________________ = 22

_________________________ = 41

Answered by Claude Tardif.
Use a 4 four times 2002-09-12
From Amanda:
The question is what is if you can only use a 4 four times and get the answer 12, 10, and 6
Answered by Claude tardif.
7+8+9+...+1000 2002-09-11
From Shirley:
My question is what is the formula for adding up numbers when you don't start with number 1? For example 3 + 4 + 5 + 6 = 18. But how could you arrive at the answer without adding all the numbers?
Answered by Penny Nom.
Subsets of the Real Number System 2002-09-07
From Christine:
In looking for a French equivalent of the word "integer", I found that the word seems not to actually exist in French, and that Canadian schools use the term "natural number" to describe what we have been trained to call "whole numbers," while using the term "whole number" to describe what we have been trained to call "integers."
Answered by Claude Tardif.
Social security numbers 2002-09-06
From Chris:
How many different number combinations are there for the social security system? We know that there are only a certain ammount of combinations available and the current US population is at 281 million. The number system should include as many nine digit combinations as possible using the numbers 0-9 in multiple different spots.
Answered by Penny Nom.
I am a 3 digit number. 2002-09-06
From Jasmine:
I am a 3 digit number. My last digit is 3 times my first digit. My firat digit is twice my middle digit. What numbers am I? There are at least 2 correct answers?
Answered by Paul Betts.
Express 5120 as a sum of consecutive numbers 2002-08-25
From Todd:
Express 5120 as a sum of more than 1 consecutive number.
Answered by Leeanne Boehm.
When is 1! + 2! + 3! + ... + x! a square? 2002-08-19
From Sarathy:
Solve :

1! + 2! + 3! + ... + x! = y 2

How do i find the solutions ?


Answered by Claude tardif.
As much greater than 47 as it is less than 105 2002-07-23
From Joe:
To find the number in question you need to find the number exactly half-way between 47 and 105. Another way of thinking about the number that is half-way between two other numbers is as the average of the two numbers given.
Answered by Leenaan Boehm.
Place six numbers around a triangle 2002-07-19
From Monika:
I need to place six numbers around a triangle, as such that one number is on each corner, and one number on each side making three numbers in one line, adding up to two hundred exactly. The number I have to use are, 40, 50, 60, 70, 80, 90. I have to use each one once, there for, each number cannot be repeated.
Answered by Claude Tardif and Chris Fisher.
Can you make up a word that adds up to 1000000? 2002-07-19
From Adrie:
My question is elementary and I am a student Please help me with the following conundrum, as it is driving me crazy.

'When the letters in the alphabet are given values from 1 to 26 for A to Z, can you make up a word that adds up to 1000000 ( one million )-used as a multiplication?'

Like 'cat' would be 3*1*20 = 60

Answered by Claude Tardif.
Square numbers 2002-05-01
From Sally:
Is one considered a square number?
Answered by Penny Nom.
a+b=10 and ab=40 2002-04-27
From April:
What two numbers add to ten and multiply to forty?

(a+b=10, a*b=40)

I think the answer includes radicals and/or imaginary numbers.


Answered by Penny Nom.
Triangular numbers 2002-04-26
From Anika:
Can you please tell me what a triangular number is?
Answered by Penny Nom.
An Olympiad problem 2002-04-23
From Aurora:
I wrote a mathematical Olympiad the other day and there was one question that I could not work out.

The question was If ab = 1, bc =2, cd =3, de =4 and ea = 6, what does a + b + c + d +e =

It was a multiple choice and the answers were : 43/6; 47/6; 49/6; 53/6; 61/6 The correct answer was 61/6, but how can do you get to that answer?


Answered by Anrei Volodin.
How many different account numbers are possible? 2002-04-18
From Andrene:
how many different account numbers are possible if the account numbers consist of a letter of the alphabet, followed by five numerical digits, and followed by another letter.
Answered by Penny Nom.
n +1, n+2, n+3, and n+4 are all composite 2002-04-09
From Jonathan:
Find the small integer n such that n +1, n+2, n+3, and n+4 are all composite
Answered by Penny Nom.
The reciprocal of a decimal mixed number 2002-03-29
From A student:
How do I find the reciprocal of a decimal mixed number?
Answered by Penny Nom.
100 from four 9's 2002-03-27
From A student:
My teacher gave us a math problem to try and figure out and I am stumped. The problem was this.... by only using four nines, how can you get an answer of 100? You can add, subtract, multiply or divide...anything you need to do ,but you can only use four nines.
Answered by Leeanne Boehm.
The square root of i 2002-03-14
From Arlene:
what is the square root of i, if i=x+yi?

what is the square root of 1-i? i'm getting problems like these in which I do not understand.


Answered by Harley Weston.
The square root of 2 2002-03-05
From Roger:
Does two (2) have a square root or do the numbers just keep going? Are there any other numbers that behave like two when it comes to extracting the square root?
Answered by Penny Nom.
Mixed numbers, whole numbers and improper fractions which are less than 2. 2002-03-03
From Dawn:
Change each mixed number to a whole number and an improper fraction which is less than 2. 3 1/2 = 2 /
Answered by Penny Nom.
What number am I? 2002-03-03
From Samantha:
I am an even number that has more ones than tens. My thousands digit is the quotient of 9 divided by 3 and my tens digit is the sum of 3 and 3. I have no hundreds. What number am I?......Please answer this for me...The answer I come up with is 3,060...Is this right?..
Answered by Penny Nom.
Alfredos house number 2002-02-21
From Aunt Patty:
Alfredos house number is between 20 and 35. The sum of the digits is less than 5. If you subtract 1 from it you would get a multiple of 3. If you add three, you get a multiple of 5. What is Alfredos house number.
Answered by Penny Nom.
Shipping charges 2002-02-19
From Sohail:
A mail order company charges a fixed fee for shipping merchandise that weighs 1 pound or less, plus an additional fee for each pound over 1 pound. If there shipping charge for 5 pounds is $4.80, and for 12 pounds is $6.20, find the fixed fee and the additional fee.
Answered by Penny Nom.
A farmer takes 100 animals to the market to sell. 2002-02-19
From Ann:
A farmer takes 100 animals to the market to sell. All are sold and $100 is earned. The prices were $5.00 per cow, $1.00 per sheep and $.05 per pig. At least one of each kind of animal was sold. How many of each kind of animal did the farmer take to market?
Answered by Penny Nom.
Square arrangements of clovers 2002-02-03
From Cassie:
Clyde had a strange fascination with numbers. One day he decided to mount his 4 leaf clover collection in groups of square numbers.he took a long piece of butcher paper and glue and began this arduous task.There was 1 clover,4 clovers.and then 9 clovers in the third set. what would be in the seventh set of 4 leaf clovers?
Answered by Penny Nom.
Baseball and sit ups 2002-01-31
From A student:
A baseball team won 8 more games than it lost. If it played 56 games, how many games did it win?

On the first day of practice, Paul did 3 sit ups. On the second day he did 5, and on the the third day he did 8, and on the fourth day he did 12. If this pattern continued, how many sit-ups did ihe do on the sixth day of practice?


Answered by Penny Nom.
Making 24 2002-01-17
From Renee:
My 4th grade daughter and I need to find a simple math sentence using 5, 5, 3, & 7 to equal 24. You can add, subtract, divide or multiple.
Answered by Penny Nom.
What is a group of three numbers called in a large number? 2002-01-15
From Laurie:
What is a group of three numbers called in a large number?

My son has this question on his 4th grade worksheet. I've taught middle school math for 7 years and nver heard of it.


Answered by Paul Betts.
Why do integers have the symbol of Z? 2002-01-11
From Stephanie:
Why do integers have the symbol of Z? Natural numbers have the symbol of N, whole numbers have the symbol of W. Our professor has asked us to find out why the letter Z is represented for integers.
Answered by Penny Nom.
A sequence of number pairs 2002-01-10
From Terry:
Three is the first number of a pair, and 8 is the second.

a. If 50 is the first number, what is the second number?
b. If 200 is the first number, what is the second number?
c. If 89 is the second number, what is the first number?
d. If a number n is the first number, what is the second number?

3-8
4-11
5-14
6-17
____
____
____
10-29
____
____
____

Answered by Peny Nom.
Number sequences 2001-11-17
From Stephanie:
My name is Stephanie. My grade 7 math class is studying number sequences and, to me, they make almost no sense at all. Here I wrote down 2 of the many problems I have for homework and was wondering if you could explain them to me (I have to find the next 3 numbers in each sequence.) It would be a big help!

10, 14, 18, 22, _, _, _.

12, 21, 39, 75, _, _, _.


Answered by Penny Nom.
How do you get to 100 by using 6 nines? 2001-11-17
From A student:
How do you get to 100 by using 6 nines?
Answered by Claude Tardif.
Some functions without numbers 2001-11-16
From A student:
I have a worksheet that is about functions. It doesn't only use numbers. I need help to figure out the function and the solution to how the answer is solved.
Answered by Claude Tardif.
Numbering pages 2001-11-15
From Lucy:
A printer uses 1008 digits to number the pages of a book. How many pages are there in the book?
Answered by Penny Nom.
Negative numbers 2001-11-15
From Jewel:
If zero means nothing or an empty set, then how can a number be less than zero, as in negative number? My understanding of negative numbers is distance from a set point in a given direction. Thus having a negative and positive side of a line is arbritary and is related to displacement rather than value. Am I in error?
Answered by Penny Nom.
what number doubled plus 20 will give you 4/3 the original number 2001-11-15
From Hank:
what number doubled plus 20 will give you 4/3 the original number

2 x__ + 20 = 4/3 of the original number

Answered by Penny Nom.
Squares of negative numbers 2001-11-03
From Susana:
I wanted to know if I can square a negative number..?
Answered by Leeanne Boehm.
Squares of one digit numbers 2001-10-15
From Needa:
What two two-digit numbers are each equal to their right-most digit squared?
Answered by Penny Nom.
eix = cosx + isinx 2001-10-10
From Peter:
Given: eix = cosx + isinx
  1. substitute -x for x to find e-ix, simplifying your answer

  2. use the given and part a to find an identity for cosx making no reference to trig functions

  3. find an identity for sinx
  4. .
  5. .

Answered by Penny Nom.
Six nines 2001-10-09
From A mom:
My middle schooler (sixth) has to calculate the integers 0-20 using only 6 nines. We have done all but the integer 14. He can not use decimals or double the nine like 99 or 19. the fraction 9/9 is okay. Keep in mind of course the order of operations.
Answered by Claude Tardif.
Building numbers 2001-10-02
From Dorothy:
Using only the numbers 1, 2. 3 & 4 and the order of operations make problems that have the answers of 1 to 50. Use each digit only once per problem. Use +, -, x, (), exponents or use 2 digits to form a 2 digit number Example 1,2=21 or 12. No Division.

We have been able to do all the problems except finding the answers of 7, 22, 31, 34, 42, 43 and 50.


Answered by Claude Tardif.
Four sequences 2001-09-24
From Nicole:
I have a few questions I couldn't figure it out. So any help will be great to find the next three numbers or letters. Here's the patters...
  1. DEGHNORY, EHILMU, HIILMTU, BEILLMRUY,...

  2. 50, 33, 25, 20, 16, 14, 12,...

  3. 1777, 1795, 1818, 1819, 1820, 1822, 1836, 1837..

  4. A,H,I,M,O,T,...

Answered by Chris Fisher and Penny Nom.
What are the two numbers? 2001-09-24
From A student:
What are the two numbers.
  1. the sum of two numbers is 15. 6 times the smaller number is 4 times the larger number .find the two numbers.
  2. the sum of the two numbers is 28. 2 times the larger number is 5 times the smaller number minus 7. find the two numbers.

Answered by Claude Tardif.
27 2001-09-10
From Kacy:
In class we were given this problem. We have the numbers 30, 29, 19,11,26. these numbers = 27 in some way but how? the numbers can be applied any way using + - / X ( ) but can only be used 1 time each. can you please help me.
Answered by Claude Tardif.
Integers 2001-09-09
From A student:
I am a Grade 9 student from Ontario Canada who has never understood how to do integers very well. I was wondering if you would be able to either send me some sites that will tell me how to calculate them in a simple way or if you would be able to write me back and give me some pointers.
Answered by Leeanne Boehm.
I have 6 digits. 2001-09-06
From Kristan:
I have 6 digits. My hundred thousands is 1 less than my 1 digit, 8 greater than my thousands digit, twice my tens digit, and 4 times my hundreds digit. My ten thousands digit is 0. What # am I?
Answered by Penny Nom.
Sharing a donut 2001-09-06
From Amanda:
You have invited 11 people over to your house one day, and your friends are hungry. You go into the kitchen and find out you only have 1 donut (with a hole in the middle). In order to feed 12 people (including you), you must cut the donut into 12 pieces with only using a straight knife and cutting 3 times. This is NOT a trick question.
Answered by Claude Tardif.
Generating uniquie 2 digit numbers 2001-07-16
From Anthony:
What methodology can I use to generate uniquie 2 digit numbers from unique 7 digit numbers. Of'course the total number of 7-digit numbers is not greater than 99.
Answered by Penny Nom.
6,000,000,000,000,000,000 2001-07-12
From James:
How would I express 6,000,000,000,000,000,000 in words?
Answered by Leeanne Boehm.
Some irrational numbers 2001-07-03
From Kellie:
  1. Is it possible to write pi as a fraction?

  2. Is it possible to write the square root of 2 as a fraction.. Explain why?

Answered by Harley Weston.
A calculation with 6 numbers 2001-06-16
From Edwin:
I'm asked to come with, and program (in Ansi -C) an algorithm that calculates all the possible results of a calculation with 6 numbers and one result. For example: I want all calculations with the numbers 3, 3, 8, 8, 2, 9, and with a result of 786. all numbers may be used once, arithmetical operations allowed are + - / *, fractions are not allowed. The problem here is what is a fast method to do this (i.e. what's algorithm that can to this).
Answered by Claude Tardif.
Four digit numbers from 1,2,3,4,5,6,7,8 2001-05-31
From Katie:
Okay.......here's my question...how many different ways can you form a four digit number out of these digits..1,2,3,4,5,6,7,8? This is how my teacher said to do this.... 8*8, 7*7,6*6,5*5,4*4,3*3,2*2,1*1. then she said to add up the products and to multiply by 7. is this correct, and if not how can you figure this out.
Answered by Penny Nom and Claude Tardif.
Three goldfish 2001-05-30
From Nathan:
A man has three goldfish. When the youngest goldfish was born, the oldest fish was three times the middle fish's age. Nine years ago the oldest fish's age was the sum of the two other fish's ages. How old are the three goldfish?
Answered by Penny Nom.
Harmonic numbers 2001-05-23
From Leslie:
The harmonic numbers Hk, k = 1,2,3.....are defined by Hk = 1 + 1/2 + 1/3....1/k

I am trying to prove by mathematical induction:

H2n >= 1 + n/2 , whenever n is a nonnegative integer.

H8 = H23 >= 1 + 3/2

Can you help?


Answered by Harley Weston.
Dividing fractions 2001-05-09
From Rina:
I just wanted to ask if you could help me in math. See I'm having a test soon and its on Dividing Fractions and I just don't get it. My math teacher says that I'll be just fine but I failed my math quiz. I went to ask eric but they could help me so they told me to go to you. So here I am asking you if you could help me.
Answered by Penny Nom.
Neither prime nor composite 2001-05-08
From Marc:
what 2 numbers are neither prime nor combitative?
Answered by Penny Nom.
24 from 2, 2, 14, and 21 2001-05-08
From Kimberly:
using the numbers 2, 2, 14, and 21. how can you get 24 using + - X or division
Answered by Claude Tardif.
The number of occurrences of 0 is __, of 1 is __, of... 2001-04-17
From Martyanne:
"In this sentence the number of occurrences of 0 is __, of 1 is __, of 2 is __, of 3 is __, of 4 is __, of 5 is __, of 6 is __, if 7 is __, of 8 is __, of 9 is __."

Each blank is to be filled with a numeral of one or more digits, written in decimal notation.

Answered by Claude Tardif.
Cube roots on a calculator 2001-03-24
From Will:
Hi my name is Will.I have a question about the calculator TI-83 Plus or the use of a scientific calculator. It is about using them to turn a cubed root to a decimal. When using that for a rational and irrational number. Rational numbers is a number that terminates or repeates. A irrational number goes on and on and uses ... (like pie). Like when you type the cube root of 8 it gives you 2, and that is a rational number. The squre root of 2 is 1.41421356... that is irrational. So why is it when you put the cube root of 16 in the calculator it says 2.5198421 that is rational it terminates at the ninth digit and my float is set for the tenth? But why when you do it by hand you get the 2 times the cube root of 2 and that is irrational? Why is that?
Answered by Judi McDonald.
Probability and a three digit number 2001-03-15
From Glenn:
A THREE DIGIT NUMBER IS SELECTED AT RANDOM. WHAT IS THE PROBABILITY THAT NON OF THE THREE DIGITS ARE PRIME.
Answered by Harley Weston.
Fibonacci 2001-02-28
From Shona:
We have been doing pattern finding with him, talking to a friend he mentioned the "Fibbinacci Series" ?, while I have tried to find a bit about it, how works etc.. what it is about, I have not really found out much, what I have I feel is way beyond him, but am still curious to know the basics of it myself. Would you be able to tell me in laymans language. Would be very much appreciated.
Answered by Claude Tardif.
T-numbers an T-totals 2001-02-27
From Mr. M. Wiseman:
Investigate the relationship between the T-total and the T-number.
Answered by Penny Nom.
Find the two numbers 2001-02-21
From A grade 6 student:
Find the two numbers:
5 is a common factor;
100 is a common multiple;
neither number is a mulitple of the other;
both are even.

Answered by Penny Nom.
Multiplying one and two digit numbers 2001-02-11
From Marty:
When multiplying do you put the number you are multiplying by on the top or the bottom. Example if you are multiplying by 6's would you write the problem 6
x 2
or 2
x 6
Maybe it just doesn't matter. But when you get to double digits, the double digit always goes on the top?

Answered by Penny Nom.
A dog and a rabbit 2001-02-09
From Josh:

1. A dog and a rabbit are 160 meters a part for every 9 meters that the dog runs , the rabbit jumps 7 meters. How many meters must the dog run in order to vertake the rabbit???

2. A cars odometer reads 15951 KM. Find the least number of kilomaters to travel until the palindromic number appears???


Answered by Leeanne Boehm.
23 + 17 * 2 / 8 + 11 = 13 2001-02-02
From David:
My child is working through basic operations, and gave this problem:

23 + 17 * 2 / 8 + 11 = 13

The teacher wants the parenthesis to be put in the right places to come up with 13.
Answered by Claude Tardif.

Subsets of the natural numbers 2001-01-30
From Christina:
How do I explain why the set of natural numbers (N) cannot be equivalent to one of its finite subsets?
Answered by Penny Nom.
14, with 5 ODD numbers 2001-01-23
From Robyn Ernst:
How can you get 14, with 5 ODD numbers?
Answered by Claude Tardif.
More repeating decimals 2001-01-17
From Alan:
I am neither a math teacher nor a student, but I hope you will consider my question anyway. I recently was discussing repeated decimals with a friend, and went on the web to find out more about a pattern was looking for years ago. In doing so I came upon your correspondence on repeating decimals.
.
.
.

Answered by Chris Fisher and Penny Nom.
Arranging some numbers 2001-01-14
From Denise:
Arrange the following numerals in two groups of four each so that each group will add to give the same sum. 1 2 3 4 5 7 8 9
Answered by Penny Nom anf Claude Tardif.
8, 4, 2, 18, 1, 9 2001-01-11
From Neyra:
Place each number below in one of the blanks to create the most meaningful and realistic story possible.
Answered by Claude Tardif.
The sum of all ten-digit numbers. 2001-01-10
From John:
Compute the sum of all ten-digit numbers. Can you determine the sum of all n -digit numbers?
Answered by Sukanta Pati.
Is n^2 - 2 a multiple of n - 4? 2001-01-10
From John:
Find all positive integers n so that n2 - 2 is a multiple of n - 4.
Answered by Sukanta Pati.
Two questions 2001-01-04
From Justine:
first: A diagonal of a rectangle is the square root of 13 meters and the area of the rectangle is 6 meters squared. What are the dimensions of the rectangle?

last: Find the ordered triple of nonzero digits (A,B,C) for which (AA)squared=BBCC where AA and BBCC are respectively, two-digit and four-digit base-ten numerals.


Answered by Penny Nom.
Why the change from commas between every 3rd digit, to spaces instead? 2001-01-03
From Clarende Duby:
Why the change from commas between every 3rd digit, to spaces instead? For example: Three million three hundred thirty-three thousand three hundred thirty-three written like: 3 333 333 instead of 3,333,333? Does it have something to do with standardizing mathematical notation worldwide?
Answered by Harley Weston.
Polite numbers 2001-01-02
From A student:
A positive whole number, greater than 1, is said to be a polite number if it can be expressed as a sum of consecutive whole numbers

Find all the polite numbers less than 21. Find a pattern in the numbers that are not polite. Explain why every odd number greater than one must be polite.


Answered by Penny Nom.
Scientific notation 2000-12-12
From Guy:
How do I express 1.000.000 in scientific notation?
Answered by Harley Weston.
Divisibility by 16 2000-12-12
From Shiling:
A number can be divided by 16 if and only if its 1st four digits can be divided by 16. How can you prove that?
Answered by Penny Nom.
A chemist had 8 flasks 2000-12-10
From Jimmy:
A chemist had 8 flasks capable of holding 12, 15, 27, 35, 37, 40, 53 and 69 fluid ounces respectively. He filled some with water and then filled all the rest except one with alcohol. He used exactly one and a half times as much alcohol as water. Which flask was left empty and which were left with water and which with alcohol?
Answered by Claude Tardif.
Two more problems 2000-12-01
From Michael:
first question:
The mean of three test grades is 74. What must a fourth grade be to increase the mean to 78?

final question:
The product of two consecutive odd positive integers added to their sum is 119. What are these two integers?


Answered by Claude Tardif.
A complex calculation 2000-11-24
From Angie:
Multiply (3-2i)2=32-2(3)(2i)+(2i)2
Answered by Penny Nom.
The magnitude of a complex number 2000-11-11
From Jeremy:
Recently, we started studying how to graph complex numbers. Our math teacher said to use what would normally be the x-axis as the real-axis and to use the y-axis as the imaginary-axis. However, when he started talking about how to calculate magnitude, that's when I became confused. For instance...
Answered by Walter Whiteley.
Expected number 2000-11-02
From James:
1)There are 40,25,50 students in the traditional,reformed,and reformed traditional Calculas sections taught by three teachers. On one fine day the three sections congregate together to watch the NOVA video tape on Fermat's Last Theorem.

a) A student is randomly selected from he crowed of students.What is the expected number of students in the same section of this student?

****this question which i did not have any idea to do as follow for the part b
b) If one of the three teachers is selected at random, what is the expected number of students taught by is teacher?


Answered by Harley Weston.
Perfect numbers 2000-10-31
From A student:
I was wondering if you could help me answer a question my pre-algebra teacher asked in class the other day. He asked if we knew what the perfect numbers where.

He told us the first number is 6 the second number is 28 but the third he did not tell us. Do you know what the third perfect number is?
Answered by Paul Betts and Chris Fisher.

Find the rule 2000-10-27
From Kelsa and Linda:
Find the rule: It had 13 in a circle then 2 blank circles then 31 in a circle then two more blank circles. Could you please help us.
Answered by Penny Nom.
A zip code problem 2000-10-26
From Rob Mathis:
Find the zip code of a place in a county so that the product of it and the zip code of another place in another county of the same name, but in a different state, is an exact multiple of the number 123456789
Answered by Claude Tardif.
Divisibility by 9 2000-10-24
From Kelera:
If the sum of the digits of a number is divisible by 9, then the number itself it divisible by 9. Why is that? How do you explain this?
Answered by Penny Nom.
Repeating decimals 2000-10-06
From Mary O'Sullivan:
I understand that with repeating decimals (those with a pattern), the number of digits repeated is put into fraction form with the same number of 9's

ex. 0.4444 = 4/9 0.145145145 = 145/999

How can I explain why this is so? Some inquisitive 8th graders are anxious to find out!
Answered by Penny Nom and Walter Whiteley.

Rounding 2000-09-14
From MaryAnn:
How many whole numbers rounded to the hundreds place, round to 400? Can you also explain why?
Answered by Penny Nom.
Rational Numbers 2000-09-14
From Josh Kuhar:
How can you tell a number is rational?
Answered by Harley Weston.
4-digit numbers using 7, 8, 9 and 0 2000-09-13
From Ryan:
Could you please help me with the following problem, How many different 4-digit numbers are there that use each of the digits 7,8,9,and 0? Please list them.
Answered by Penny Nom.
How many 17's and 19's total 1000? 2000-09-07
From Jonathan:
My question is: what 2 numbers would multiply 17 and 19 for a total of 1000. The numbers should not contain any decimal.
Answered by Penny Nom.
n3 + 2n2 is a square 2000-09-04
From David Xiao:
determine the smallest positive integers, n , which satisfies the equation

n3 + 2n2 = b

where b is the square of an odd integer


Answered by Harley Weston.
A number problem 2000-09-02
From Lalita Balu:
Can you arrange 1234567890 to add up to 100?
Answered by Penny Nom.
Three consecutive odd integers 2000-08-18
From Wallace:
A six-digit integer XYXYXY, where X and Y are digits is equal to five times the product of three consecutive odd integers. Determine these three odd integers.
Answered by Paul Betts.
Five times a cube equals three times a fifth power 2000-07-05
From Harman Chaudhry:
Which is the smallest 10-digit number to be five times the cube of one number and also three times the fifth power of another?
Answered by Penny Nom.
Calculator and telephones keypads phones 2000-06-27
From Thomas Smith and Veronica Yates-Riley:
On a keyboard, why is the number "7" on the top of the keypad as opposed to the number "1" at the top a telephone keypad?
Answered by Penny Nom.
Central Limit Theorem and Law of Large Numbers 2000-06-26
From Jonathan Yam:
The Central limit Theorem states that when sample size tends to infinity, the sample mean will be normally distributed. The Law of Large Number states that when sample size tends to infinity, the sample mean equals to population mean. Is the two statements contradictory?
Answered by Paul Betts and Harley Weston.
A bar over a Roman numeral 2000-06-21
From Charles Fontaine:
Is there any significance to the bar placed over some Roman numerals?
Answered by Penny Nom.
Names for large numbers 2000-06-16
From Mireille Carthier:
I am trying to find the common names (in English and French) for the big numbers as follows:
English French
106 million million
109 billion milliard?
1012 trillion billion?
1015 ?? trillion?

Thank you very much


Answered by Claude Tardif and Harley Weston.
Problems 2000-06-06
From Debbie Cummins:
I am a Mum of a 12 yr.boy needing help with some math problems. I need not only the answers but how it is worked out.
  1. Both the leftmost digit & the rightmost digit of a 4 digit number N are equal to 1. When these digits are removed, the 2 digit number thus obtained is N div by 21 Find N.
  2. Find all 3 digit even numbers N such that 693xN is a perfect square, that is, 693x N = k x k where k is an integer.
.
.
.

Answered by Paul Betts and Claude Tardif.
Percents with mixed numbers 2000-06-04
From Julie:
How would you find the percent of 33 1/3 out of 90 by using fraction settup!I can do whole numbers but not mixed. This is how to do whole numbers say it was 33 of 90

33 x 90 = 2970
100 x 1 = 100
2970 divided by 100
the answer is 29.7

so how do you do it with a mixed number


Answered by Penny Nom.
Powers of i 2000-05-24
From Paul Fieldhouse :
What is the result of raising i to the googol power? is there a rule or pattern to raising i by increasing powers of 10?
Answered by Penny Nom.
The square root of -1 2000-05-19
From Gary:
i am not a student i am just some one that heard something and i can't be sure on the answer...my ? is what is the square root of -1? i think it is -1 but not sure can you let me know please thank you
Answered by Harley Weston.
An expected value 2000-04-24
From Carl Pride:
Suppose that in a statistics class of size 23, each student has a probability of passing of 73 percent.

sample:

What is the expected number of students who will pass??


Answered by Harley Weston.
Natural numbers 2000-04-12
From Sara:
What is a natural number???
Answered by Penny Nom.
How many zeros? 2000-04-09
From Greg Potts:
The natural numbers 1 to 25 are multiplied together (1 x 2 x 3x..24 x 25). How many zeros are there in the product of this multiplication?
a)6 b)7 c)5 d)10 or e)4?

Answered by Harley Weston.
root(-1)* root(-1) 2000-03-20
From Michael Moran:
i squared = -1

but

i squared = root(-1)* root(-1)
= root( -1*-1)
= root(1)
= 1
-1 doesn't = 1

can you help me with my question


Answered by Claude Tardif.
Six digit numbers using 1,2,5,6,7, and 9 2000-03-20
From Rachel:
How many different six-digit numbers can you make using the digits 1,2,5,6,7, and 9? How many of these six digit numbers are divisible by six?
Answered by Claude Tardif and Denis Hanson.
Rationals, irrationals and integers 2000-03-14
From Erin McKeon:
Why does the letter J represent the set of integers, the letter Q represent a set of rational numbers and the letter P represent a set of irrational numbers? What do each of these letters stand for?
Answered by Harley Weston.
Triangular numbers and square numbers 2000-03-02
From Emily McCallum:
Would someone find me some math activities using triangular numbers and square number. Especially acitivities that actually form the shapes. They need to be at the 5th grade or 6th grade level. But you can not find anything that can be tauht at these level, that fine. I just need to be able to teach this new subject or kind of math to my kids.
Answered by Rick Seaman and Penny Nom.
More on googols 2000-02-23
From Kevin Brennan:
Could you please give me a couple of examples of when you MATH guys would use a google. Do youu measure, like, galaxies with it ? Or was it created to keep track of Mr Gate's bank account.
Answered by Penny Nom.
Number sense 2000-02-17
From A college student:
I am a college student and I am to answer the following question: Describe ways of telling by simple inspection when a fraction has a value close to 0; close to 1/2; and close to 1. I am to imagine that I am a 3rd grader answering this question. Any thoughts please?
Answered by Penny Nom.
Compatible numbers 2000-01-27
From Angie:
Use compatible numbers to estimate each product and quotient.
23*1/2
1/3*11

Answered by Penny Nom.
Factors 2000-01-27
From Pam:
My son [age 9, 4th grade] has a math problem with factors. I need to know how to do the problem that involves the factor 24. I'm not even sure where to start.
Answered by Harley Weston.
A three digit number 2000-01-13
From Manali Shah:
My son is in 5th grade, he has a math problem. There is three digit number, which is greater than 500, can be divided by 7,9 and 11, but cannot be divided by 2,4,6, and 5.
Answered by Harley Weston.
Phone Number Possibilities 2000-01-09
From Hamilton Weston:
How do phone companies arrive at the possible number of phone numbers that can be generated for customers?
Answered by Harley Weston.
Fibonacci Numbers 1999-12-15
From Gary Nelb:
I'm doing a project on fibonacci numbers and I'm using different starting values and finding out if different starting values to see whether or not the ratios still get closer to phi. I was wondering, what numbers should I use. Should I use two of the same # like 2 and 2, or numbers like 1 and 2, or even something totally different.
Answered by Denis Hanson.
Bases other than 10 1999-12-06
From Garret Magin:
We are doing a lesson on numbers of other bases than 10. We are working with binary, octal, and Hexadecimal. I was wondering what is used to represent number of different bases other then 16? Does it just continue on with the alphabet and if so what happens when you get to Z. It would be a help if you could answer this because it is really bugging me. And none of the math teachers at my school could let me know.
Answered by Claude Tardif and Patrick Maidorn.
Whole numbers 1999-12-02
From Patty:
I'm in a 5th grade class, please explaine what whole numbers are.....
Answered by Penny Nom.
Critical numbers 1999-11-17
From Sarah:
My question is this:

If f(x)=2x3+4x2-9x+8

and the derivative is f'(x)=6x2+8x-9,

how do I find the critical points if f'(x) is not factorable??
Answered by Penny Nom.

Number sequences 1999-11-11
From Justin Schessler:
I CAN NOT figure out this sequence or how we continue this sequence...

3,2,6,5,15,14,___,___,___
Answered by Chris Fisher and Harley Weston.

Crossing number 1999-11-06
From Christian:
The crossing number of a graph G, denoted cr(G) is defined to be the minimum number of (pairwise) crossings of edges among all drawings of the graph in the plane. For example, cr(K5)=1 and cr(K3,3)=1.
What is cr(K7,7)?

I figured out that the answer is 81.

Now I am trying to figure out if K7,7 can be drawn in the plane with less than 81 crossings?

I'm not sure how to approach this one. Other than actually drawing it out and checking by trial and error, I am not sure how to approach this problem. Please help!
Answered by Denis Hanson.

Nines and ones 1999-11-05
From Greg Miller:
Using the numbers 1, 1, 9, 9, only once each, how can I create an expression that equals 10 using only the basic functions of addition, subtraction, multiplication, division, and/or parentheses?
Answered by Claude Tardif.
Three numbers 1999-11-03
From Walter:
Three whole numbers , when added two at a time,yield the sums 75, 111, and 128.

Find the smallest of these three whole #'s.
Answered by Penny Nom.

An odd number of factors 1999-10-22
From Melissa:
What is the common name used for numbers that have an odd number of factors? What is the least positive integer that has exactly 13 factors?
Answered by Penny Nom.
Find the missing numbers 1999-10-12
From Bonnie:
I must find the missing numbers and function rules

noutput
710
1220
3940
2430
16b
87c

we are totally confused can someone help
Answered by Denis Hanson.

A sum of two squares 1999-10-08
From Marksmen:
what is the smallest whole number that can be written two ways as a sum of two different perfect squares? i.e.11squared plus 3 squared is 121+ 9=130 and7 squared + 9squared=49 +81= 130. Are there any smaller? I am stumped!
Answered by Claude Tardif.
Palindrome 1999-10-06
From Chris Ware:
What is a non paladromic number?
Answered by Claude Tardif.
Numbers with the digit 2 in 1...1000 1999-09-20
From Jessica:
Is there a trick to finding out how many numbers containing the digit two is there from 1 to 1000?
Answered by Walter Whiteley.
2,5,8,1 yields 24 1999-09-12
From Terry Hill:
I was wondering how do I make the numbers 2,5,8,and 1 equal 24?
Answered by Penny Nom.
The sum of the cubes is the square of the sum 1999-08-25
From Bernard Yuen:
How to prove 13 + 23 + 33 + 43 + ... n3 is equal to (1+2+3+...n)2? (for n is positive integer)
Answered by Harley Weston.
Evens and odds 1999-05-04
From Emily Hays:
My name is Emily Hays, I am in the 6th grade and my teacher gave me some extra probelms to see if I could solve them. She can't find the KEY and I can't figure out the probelm so she suggested I e-mail you guys to see if you could help us. Here's the Problem:

The numbers 1,2,3...,1999 are written on the blackboard in the classroom. Evertime the teacher enters the room he chooses two numbers on the blackboard, say a & b, with a> or equal to b, then he erases them and writes the difference a-b somewhere on the blackboard. After this procedure is carried on 1998 times, there will be only one number left on the blackboard. Prove the last remaining number must be even.

I hope you can help us!
Answered by Chris Fisher and Harley Weston.

Large Numbers 1999-04-05
From CK:
Hello. I need a lesson plan for grades 4 to 7 dealing with the concept of large numbers. Specifically, how to teach scientific notation. The lesson plan has to deal with the following: the size of the universe is so huge that is is almost beyond the ability of the mind to comprehend. One way is to measure...
Answered by Chris Fisher.
T-shirts 1999-04-04
From Valerie:
At a discount T-shirt shop the manager marked a special batch of T-shirts that originally sold for $2.00. When all the shirts sold the shop had collected $603.77.

How many shirts did they sell and what was the price per shirt?
Answered by Jack LeSage and Penny Nom.

Complex numbers/polar coordinates 1999-03-25
From Kate Cegelis:
What is the relationship between complex numbers and polar coordinates?
Answered by Harley Weston.
The square root of two is never supposed to end 1999-03-06
From a wondering student:
i am algebra II and am in the 9th grade. today we were talking about rational and irrational numbers. When we were talking about square roots my friend and i were talking and we thought of something. if you have a square with sides of length one then the diagonal of the square is the square root of 2. Now the square root of two is never supposed to end. But the diagonal of the square ends so therefore doesn't the square root of 2 end. our math teacher did not really answer our question because it was not in the lesson plan and not to many people would see where we were coming from. the answer is really bugging me and i would like to have your input.
Answered by Jack LeSage and Penny Nom.
The Board Problem 1999-02-15
From Avery:
Mr. Avery has 3-foot boards and 4-foot boards. If he puts the 3-foot boards in a line, they have the same length as the 4-foot boards put in a line. Altogether he has between 16 and 25 boards. How many 3-foot boards does he have?
Answered by Jack LeSage and Penny Nom.
Magic Squares 1999-02-11
From Katie Powell:
My name is Katie Powell. I'm in the 7th grade, taking Algebra. I live in Houston, Texas. My problem is this:

"Use the numbers 1-9 to fill in the boxes so that you get the same sum when you add vertically, horizontally or diagonally."

The boxes are formed like a tic-tac-toe -- with 9 boxes -- 3 rows and 3 columns.

Can you help?
Answered by Jack LeSage.

Grade 4 curriculum 1999-02-09
From John:
My daughter is in grade 4 and her math teacher has asked her to write out the number 1 to 10,000. She is currently at 4567 and is sick of it. When I complained that this was not a usefull exercise she said that the Ministry of Education curriculum requires this and gave a photocopy of the page. It states: "read and write whole number to 10,000 in standard, expanded, and written forms (eg., 9367=9000+300+60+7 = nine thousand three hundred sixty-seven). Has the teacher interpreted this the wrong way?

Thanks John
PS. Do you have a web address for Ontario Math and Science Teachers organization?
Answered by Jack LeSage.

Cannonballs 1999-01-27
From Roger King:
How many cannonballs can be stacked in a triangular pyramid?
Answered by Penny Nom.
Absolute value of i 1999-01-06
From Wayne Bagley:
I would like to know what is the absolute value of i. I need an answer suitable for the secondary level.
Answered by Harley Weston.
Place Value 1999-01-05
From Reg:
I work in a Section 27 classroom with various grades. I am looking for assistance on how to deliver a lesson on place value:decimal numbers keeping in mind that they need to see it (visual) to comprehend. I am looking for grade 4-6 level.
Answered by Jack LeSage.
Complex numbers and the quadratic formula 1998-12-25
From Richard Peter:
My age is 16, and my name is Richard. My question relates to the topic complex numbers & the quadratic formula.

I would like to know how to solve quadratic equations in which the discriminant is less than 0 (i.e. we get two complex solutions to the quadratic)

3x2+2x+5 = 0

and how mathematicians like euler contributed to this field. If it would be possible I would also like to know how this type of quadratic equations can be graphed?
Answered by Harley Weston.

Complex Numbers 1998-12-23
From Wayne Bagley:
I would like to know what is the square root of i , and i squared? I am looking for a response appropriate for secondary level students.
Answered by Harley Weston.
6.99999... = ? 1998-12-05
From Tom:
I have had a rather heated arguement with my students. Please settle this for me.

Solve <,>, =
6.99999... __ 7

Thank you.
Answered by Penny Nom.

Pick any odd number, square it, and then divide it by 8 1998-11-27
From Brenda Meagher:
Pick any odd number, square it, and then divide it by 8. No matter what odd number is chosen and squared and divided by 8, the remainder is one.

Could you please explain this to me or is there a pattern that I am not aware
Answered by Harley Weston.

A number between 1998-11-25
From Jimmy Nikola:
A number is less than 3,425 + 8,630 and greater than 7,614 + 4,429. What could the number be?
This question is on page 93, question number 23. in Silver Burdett Ginn Math book, which is a 3rd grade math book.

Answered by Jack LeSage.
Triangular Numbers 1998-10-30
From Matt:
i would like to know about triangular numbers and it history i would also like to know about the history of prime numbers thank you
Answered by Chris Fisher.
A Six Digit Number 1998-10-27
From Craig Bedard:
If a certain six-digit number is split into two parts, one constituting the first three digits and the other the last three digits, and the two parts are added and the resultiong sum squared, it is found that the product is the original six digit number. What is the original six-digit number?

At first it seemed liked an impossible question, until it hit me...how long will it take you?
Answered by Jason Stein and Dan Usselman.

A Number Trick 1998-10-26
From Brenda Meagher:
I'm a 37 year old mother and i have returned to school for personal reasons. I was faced with this question.
Choose any number less than 10, multiply it by 8547, then take your answer and multiply it by 13. I chose the number 8.

8547
x 8
--------
68376

and

68376
x 13
---------
205128
68376
---------
888888

Any number chosen from 1 to 9 will result in the same form of answer.If the number two is used you will end up with 222,222.etc So my question is, is there another number that will results in the answers.
Answered by Harley Weston.

Adding Fractions 1998-10-15
From Lindsay:
please help me!!!!
Answered by Harley Weston.
Composite Numbers 1998-10-07
From Greg Murphy:
What type of composite number has an odd number of factors?
Answered by Harley Weston.
Scientific Notation 1998-10-07
From R. Pierce:
Help! How do you express the following numbers in scientific(exponential)notation:0.00036 and 2998000000. I understand how to solve them if it was an equation, but this is confusing me. Please help me answer my questions. Thank you.
Answered by Harley Weston.
Number Systems 1998-10-07
From Frans:
What other number system are there besides the real number system and the complex number system?
Interested adult
thanks

Answered by Penny Nom.
Help the Helper 1998-09-20
From John Derr:
I am TRYING to help my niece with her Community College Teacher's math course! I have a BS and an Engineer yet stumble on what her book is asking:

  1. Fill in the blanks to continue this sequence of equations:
    1=0+1
    1+3+1=1+4
    1+3+5+3+1=4+9
    _______________=____
    _______________=____
  2. What expression, suggested by part (1.), should be placed in the blank to complete this equation?

    1 + 3 + 5.........+ (2n-3) + (2n-1)+ (2n-3)+...+5 + 3+ 1=_________


Answered by Penny Nom.
Five Factors 1998-09-19
From Derek Yau:
To whom it may concern,

I have difficulty in getting the solution to the following question:

Find 5 numbers that have exactly 5 factors.

I got 16, 81 but couldn't find the rest. I believe that in order to have 5 factors, it has to be a square number. Isn't it true? I guess there may be a pattern to this.

Thanks for your help.
Derek Yau.
Answered by Penny Nom.

Rational and Irrational Numbers 1998-09-19
From Ri:
I am trying to explain rational & irrational numbers to my niece who is grade 7 and am having difficulties. Could you please explain the difference between rational & irrational numbers.

Thank you
Ri
Answered by Penny Nom.

A Pattern 1998-09-14
From Krystin:
Question:
1=1x1
1+2+1=2x2
1+2+3+2+1=3x3

draw a series of diagrams to illustrate the above pattern
Answered by Penny Nom.

Rational Numbers 1998-09-05
From Kristin Mckenzie:
Iam a secondary student with a math question I don't know how to do.
This is the question:

Explain whether each decimal number represents a rational number.

(a) 0.16

(b) 0.12

(c) -3.125

(d) -0.27

(e) -0,212 112 111 2 ....

(f) 0. 457 92

If it wouldn't be a problem i would really appreciate it if you sent back the instructions on how to do the question.

My name is Kristin Mckenzie and my return e-mail is lexus1999@hotmail.com

Thank-you for you time
Answered by Jack LeSage and Harley Weston.

Two Problems 1998-07-28
From James Pulver:
How do you solve these problem? If log abc=16 and log ac=12 , find b. (The logs are log base 10.)
and
If a and b are real numbers, i^2 = -1 and (a+b)+5i=9+ai what is the value of b?

Answered by Jack LeSage.
Social Security Numbers 1998-07-07
From Valerie Lucas:
How many social security numbers are possible?

the answer is 10^9

why isn't the answer 9^9?
Answered by Harley Weston.

Roman Numerals 1998-02-05
From Dawn Hilgenberg:
How would I read a Roman Numeral such as LXII? Does the L in front make the X equal a lesser amount? We have been discussing how to read these and I can find no information about how to read an extended Roman Numeral problem. Help!!
Answered by Harley Weston.
Divisors of 6n 1998-01-26
From James:
If n is a positive integer such that 2n has 28 positive divisors and 3n has 30 positive divisors, then how many positive divisors does 6n have?
(a) 32 (b) 34 (c) 35 (d) 36 (e) 38

Answered by Penny Nom and Haragauri Gupta.
The two-digit numbers from 10-99 1998-01-21
From Alexis Riddle:
My name is Alexis. I'm a student in 8th grade and I'm taking Algebra I Honors. My question is: In a string of numbers, two adjacent digits are considered as a two-digit number. For instance, the string 11012 contains the numbers 10, 11, and 12. What is the number of digits in the smallest string that contains all of the two-digit numbers from 10-99? Please help and thank you for any assistance you can give me.
Answered by Penny Nom.
Find the Numbers. 1997-12-28
From Sandy C:
The sum of the squares of two numbers is 53. Twice the greater, minus the lesser is 11. Find the numbers.
Answered by Penny Nom.
(-2)^x 1997-12-12
From Ken Bokinac:
During our math class last week we came accross an equation that we could not graph we were wondering if you could help us try and graph it. The question is:

y=(-2)^x
Answered by Harley Weston.

Driving me CRAZY. 1997-11-17
From Billy Tran:
Write a two digit number (both digits different and neither equal to zero) and then express the same number but now composed of the same two digits in reverse order with some mathematical symbol (+, -, *, /, roots, !, or exponents). for example: 25 = 5^2. Now give me another.

Hope you get this, cause its driving me CRAZY
Answered by Penny Nom.

Negative Primes. 1997-11-10
From Leah Zucker and Paul Michael:
I have received the following question via e-mail from my granddaughter: "Can negative numbers (like -7) be prime? If not, why not?"
Answered by Chris Fisher.
Multiplying imaginary numbers. 1997-11-03
From Jim Catton:
Here is the question:

(square root -2) x (square root -8)

My algebra suggests two possibilities .
.
.

Answered by Walter Whiteley, Chris Fisfer and Harley Weston.

Billions and more! 1997-09-15
From Mahabir B. Gupta:
I would like to know how you americans write the number 1 billion.

Do you say "One thousand million"..can you answer by giving me examples?

1,000,000----> 1 million
1,000,000,000---->1 billion

Why is it that in spanish it is different?
Answered by Penny Nom.

Triangular Numbers. 1997-09-08
From Rachel:
Show visually that the square number 16 is the sum of two triangular numbers. Which two?
Answered by Penny Nom.
Prime Numbers 1997-09-04
From Jack Bedard:
Hi. I have a real stupid question

What are prime numbers?

My name is Jack Bedard and I am in grade 5.
Answered by Penny Nom.

When is ( n^3+1)/(mn-1) an integer? 1997-02-11
From Ronald Lui:
Determine all ordered pairs (m,n) of positive integers such that ( n^3+1)/(mn-1)is an integer.
Answered by Richard McIntosh.
Repeating Decimals 1997-01-24
From Grant Reed:
Is there a way to tell that the repeating decimal for 1/17 has no more than 16 repeating digits?
Answered by Penny Nom.
Term definitions 1996-09-18
From M. Seltzer:
A 7th grade algebra student would like the definition of the terms proper factor and perfect number.
Answered by The Centralizer.
How do you find out if a number is a composite or prime? 1996-09-16
From Trish Feenstra:
Hi my name is Trish Feenstra. In grade 8. How do you find out if a number is a composite or prime if it is big like 37 529? Reply soon got a test on Friday the 13th
Answered by Denis Hanson.
Thousands, millions, billions and trillions 1996-08-22
From Blaine:
What are the names of the periods in groups of numbers like 123,456,789. I need to know the names of them for school. I already know the first ones: units thousands millions billions trillions.
Answered by Penny Nom and Diane Hanson.
Pronunciation 1996-03-05
From Todd Stienhauser:
Is 123.4 pronounced "one hundred and twenty three and four tenths"or "one hundred twenty three and four tenths"?
Answered by Penny Nom.
Prime numbers 1996-02-22
From Charles Hewitt:
I would apprecaite it if you could share with me any algorithm to determine whether a number is prime.
Answered by Penny Nom.
Complex numbers 1995-10-22
From Jacquie:
Why should we teach complex numbers in high school?
Answered by Harley Weston.
2^(2^1954) est-il premier? 2005-10-04
From Un eleve:
2(21945)+1 est-il premier?
Answered by Claude Tardif.
Avec les chiffres 1 3 4 6... 2005-04-14
From Ambre et Miel:
avec les chiffres 1 3 4 6 utilisé une seule fois chacun soit en div multipl addit ousoustr je dois avoir 24 comme résultat
Answered by Claude Tardif.
Combien y aura t il de 9 dans la réponse? 2000-06-17
From Marie:
J'ai lu une énigme mais je n'ai pas la réponse."si l'on divise 123 456 789 par 999 999 999, combien y aura t il de 9 dans la réponse?"
Answered by Claude Tardif.
 
Page
1/1

 

 


Math Central is supported by the University of Regina and The Pacific Institute for the Mathematical Sciences.

CMS
.

 

Home Resource Room Home Resource Room Quandaries and Queries Mathematics with a Human Face About Math Central Problem of the Month Math Beyond School Outreach Activities Teacher's Bulletin Board Canadian Mathematical Society University of Regina PIMS